AAMC CARS Diagnostic2 [Ext]

The author begins the passage by first mentioning the two perspectives on how literature developed: “One group views social change as the catalyst for both technological and literary developments; another argues that technology leads the evolution of both society and literature.” But in the next few paragraphs, the author brings up thinkers who argue against that second view. Such is first signaled by the beginning of paragraph 2: “Yet Watt’s account of the rise of the novel stresses not technological change but the growth of a reading public as fundamental to the eighteenth-century literary innovation.” While the author argues against the idea that technology comes before social and literary innovations, the author also argues that technology clearly plays a part in those developments; it just is not the main or single driving force behind them. That is crystallized by the end of the passage: “One undeniable truth is that technological and scientific discoveries never take place in a social vacuum [… there is every likelihood that scientific inquiry is directed in complex ways by social issues and the needs of those in positions of power.” So I’m looking for an answer choice which states this main idea: that literature begins through a complex interplay between social and technological forces.
A is a perfect match for my prediction. Though the author focuses more on arguing against the idea that technology drives innovation, the author also freely admits that technology plays a role in those innovations. After all, as the author states in the first paragraph, the novel is “perhaps the only literary genre that is unequivocally a product of the medium of print,” or a technological innovation. A has to be right.
B is meant to be the attractive wrong answer choice here, because it plays into what feels like the argument in the middle paragraphs: where the author argues against seeing technology as the primary driver of literary developments. But that actually suggests why B has to be wrong: because at no point does the author admit that any changes in literature came from technological factors. Even the novel, which the author claims only exists because of the technology of the printing press, is described in paragraph 2 and 3 as the result of a social change (the growth of a reading public) and the technological change of the printing press. So B can’t be right.
The passage brings up many experts on literary changes, but there is no real disagreement between them. All those cited argue against what Williams in paragraph 5 calls “technological determinism,” so C has to be wrong.
D has to be wrong because it introduces a term that is not in the passage. The author focuses on the relationship between social and technological change on literature; the literary developments discussed in the passage are associated with the complex interaction between those two forces, so there is no sense in the passage that developments within literature are overshadowed by social and technological change.
All I need to answer this question is to confirm Watt’s position. My initial reading of paragraph 2 tells me that it was meant to argue against the claim that ended paragraph 1: that perhaps “technology leads the evolution of both society and literature.” Paragraph 2 is so short that it just can’t easily deviate from that work, so I’m looking for an example which indicates that “the growth of a reading public” was what helped encourage the novel to emerge.
I can eliminate A because there is no necessarily strong relationship between a work’s subject matter and its audience. Since A doesn’t talk about an expansion in the reading public, it has to be wrong.
Again, B doesn’t talk about the growth of a reading public: of people who read, not just people who wrote. So B has to be wrong.
Though C doesn’t explicitly mention reading, it has to be correct. The preconditions to create a populace that reads are education (in order to help people become literate) and leisure: to have the time to read. If C were true, then it would support Watt’s point by stating that those preconditions were in place.
I know that Watt’s point argues against technological change, not social change, so D has to be wrong.
This is a straightforward weakener, but one that is luckily phrased in such a way that I know it’s asking me about the main idea. So as I evaluate the options, I need to keep in mind that the author is arguing against technological change as the primary driver of literary developments, which means the correct options will say something about how technology did drive developments.I. I actually would support the author’s argument, since it describes a social change—the “public appetite for fiction”—as driving technological innovations. That challenges the notion that technological change was a primary driver of literary development, so I has to be wrong.

II. II, though, argues that a technological development—the development of the printing press—encouraged the social conditions that helped change literature, so II is correct.

III. Wealth here is not a technological development, but a social one, so III is in in line with the passage argument. That makes III incorrect.

This matches my prediction, so I can move on.
III is incorrect because it does not describe a technological development driving literary change, but rather a technological development driven by social change. So B is wrong.
I is incorrect, so C has to be incorrect.
D is wrong because both I and III are wrong.
Though the author mentions this term in the middle of the passage, it is present throughout the entire passage. The terminology echoes the opposition the author mentions in the first paragraph: specifically the second term of that opposition, that some argue “that technology leads the evolution of both society and literature.” I can confirm this by checking the mention of the term in paragraph 5. When the author claims that “Williams also opposes the idea of technological determinism,” the author explains this opposition by stating that “In no way did the new medium create social conditions.” So I’m looking for an answer choice which claims that technological determinism means technology creating social conditions.
The passage is not focused on discussing the reception of revolutionary inventions; even if I stretch the definition of literature to include literary developments, “technological sophistication” is such a vague term that is not discussed much in the passage that I can eliminate A. I need an answer choice which says that technology itself, and not just a society’s orientation towards that technology, drives social change.
B is the opposite of what I’m looking for, since it claims that technology is driven by social changes. So B must be wrong.
The right answer needs to mention both technology and society in order to suggest that tech drives social change. Since C doesn’t mention society at all, it has to be wrong.
D is last, and it is also correct. It matches my prediction: that technology drives social change.
The author’s main idea is that tech does not drive social change. So the author would not agree that the technology of television is a primary driver of today’s culture. That means I’m looking for an answer choice which somehow suggests that today’s culture is the result of television, but not just or primarily as a technology.
A sounds pretty good because it focuses on social change instead of technology. But A still leaves room for the idea that technology is ultimately to blame for our current culture, since it does not talk about the creation of television, but its “technical refinement.” Since A still allows for the possibility that the technology of television may have played some role in the social changes mentioned, I have to eliminate A. I also know that I’m now looking for a choice that mentions the creation of television.
B is almost the same as A, but talks this time about the need for television as opposed to the refinement of television. That makes B the right answer.
The author would argue against C, since the author argues against the idea that technological change drives social change.
D has to be wrong because, despite being oddly worded, it still suggests that technological change drives social change: in this case, by reducing the social significance of television.
The question stem describes a technological change in a medium that does not transform the medium. That is definitely in line with the passage argument about how tech does not drive culture by itself: that there are elements which are not simply the result of technology. So I know I’m looking for an answer choice which states that the new information in the question stem supports the main idea.
The question stem describes a technological change in a medium that does not transform the medium. That is definitely in line with the passage argument about how tech does not drive culture by itself: that there are elements which are not simply the result of technology. So I know I’m looking for an answer choice which states that the new information in the question stem supports the main idea.
works. I can see how the situation in the question stem describes how technology (recording technology) does not determine the form taken by literature (the retained style of traditional oral stories). And since that is in line with the main idea, I would choose B and move on.
This is incorrect because while literature is alluded to in the question stem when it mentions printed transcripts, the information in the question stem does not have anything to do with the marketplace.
I have no clue what the public demands in the situation described in the question stem, so I can eliminate D.
This is a passage check question; I need to determine what support, if any, is given for the claim referenced in the question stem. Eisenstein’s mention in paragraph 6 is a single quote that occupies the entire paragraph. There is just no room for the claim to be supported by evidence, since I know that the next paragraph sums up the passage argument. So the right answer has to say that the claim is not supported in the passage by verifiable evidence.
I can eliminate A because the right answer has to start with a “No,” not a “Yes.”
B has the same problem as A, and so must also be incorrect.
C has to be correct because it runs counter to the passage argument. Though the author argues that technological development is not the primary driver of social change, at no point does the author deny that technological developments like printing shops had an effect on cultural developments.
D matches my prediction, since it is a long way of saying that the Eisenstein quote is provided with no evidence to support it.
The passage lists 4 kinds of changes in paragraph 2. Genetic changes are “intergenerational changes,” growth changes are “ontogenetic changes-i.e., relatively irreversible changes occurring within an individual’s lifetime,” while physiological changes are described as reversible: like moving from one location to another. Behavioral changes are discussed in paragraph 3 as changes in behavior. Allen and Bergmann are discussed in paragraph 4, where they both are used to account for why “The human body comes in various shapes and sizes.” They both suggest that climate accounts for why some people are taller or shorter. Both are implied to explain how differences in human forms are responses “to long-standing adaptations to certain habitats.” Only genetic changes would make sense on such long timescales, so that’s what I’m predicting the right answer will be.
Behavioral changes wouldn’t obviously affect one’s body size in the ways that Allen and Bergmann describe. The author states in paragraph 4 that while “some variation is undoubtedly due to nutrition,” which could be read as either a behavioral or physiological condition, but is ultimately more interested in exploring those responses “to long-standing adaptations to certain habitats.” Since behavioral changes are the most rapid form of change, A does not fit what I’m looking for, so I can eliminate it.
B runs into a similar problem to A: its effects are too rapid, and too locked into an individual’s lifespan, while Allen and Bergmann are talking about changes that would take generations to solidify. B is therefore wrong.
C is closer, but is still too locked into an individual lifetime to qualify as “long-standing adaptations to certain habitats.”
D matches my prediction, so I can choose it and move on.
This speaks to the main idea of the passage. The author introduces the four kinds of changes in order to hone in on behavioral responses in paragraph 3. There, the author claims that behavioral change “has proven to be a highly efficient means of responding to environmental stress, permitting Homo sapiens to explore many possibilities for moving into new environments.” So the answer has to say something about behavioral changes.
This has to be incorrect because the author states that it is our “culturally patterned ways of dealing with environmental stresses” (paragraph 3), not genetic changes, so A has to be wrong.
B too has to be wrong because it is a psychological, rather than physiological, benefit which explains our ability to inhabit diverse environments.
C too is incorrect because it does not refer to behavior.
D, however, does mention behavioral changes, so it has to be correct.
This sounds like a reference to Allen and Bergmann in paragraph 4. A quick glance at that paragraph tells me I need to pay attention to Allen, since he is the one who talks about extremity size. Allen says that “extremities and appendages […] tend to be shorter in colder climates,” and so extremities and appendages will be longer in warmer climates. The evidence in the question stem would therefore challenge Allen’s rule, since Allen doesn’t mention cases in which some extremities are long and some are short.
A definitely claims that there may be problems with Allen’s rule, so I can choose A and move on.
B is definitely here to test if I’ve read paragraph 4 closely. I know that Bergmann doesn’t talk about extremity size, so I can eliminate B.
C is also an attractive answer choice, because it too suggests that there are problems with Allen’s rule, but it goes further than the evidence mentioned in the question stem can support. At least part of Allen’s rule is correct, since the members of the population do have long upper extremities. So I can eliminate C.
I can eliminate D for the same reason I eliminated B: because Bergmann isn’t invoked by the evidence in the question stem.
Drawbacks to Allen’s and Bergmann’s rules can only be mentioned after the rules are mentioned. The last paragraph begins with some factual information about then, but then states that “Even now no one argues that these rules can be strictly applied to animal populations in all cases; however, they are possible explanatory models for various adaptive traits.” The only drawback I can see there is that the rule cannot “be strictly applied to animal populations in all cases,” so that’s what the right answer has to say.
This information precedes the actual explanation of Allen’s and Bergmann’s rules in paragraph 4, but it’s just factual information; there’s no obvious drawback to the rules being formulated when they were. So I can eliminate A.
B is synonymous with the claim that Allen’s and Bergmann’s rules cannot “be strictly applied to animal populations in all cases,” so I can choose B confidently.
Though C is in the right place—since it follows Allen’s and Bergmann’s rules—it has a similar problem to A. C is just factual information, with no obvious downside, so I can safely eliminate it too.
The end of the last paragraph says the opposite of D: that these rules “are possible explanatory models for various adaptive traits.” Since D goes against the passage, it has to be incorrect.
Once again, this question set is asking me to decide between the four kinds of changes mentioned in paragraph 2. I know that this has to be a short-term change, since I sweat in reaction to heat that is present, so genetic responses are out. Sweat is not a behavior—there is no sense of decision to it—so behavioral change is also out. And sweat is a reversible response; I’ll stop sweating if the temperature changes, or if I move to someplace where the temperate changes, so sweat can’t be a growth change. That leaves physiological changes, which makes sense, since this is reversible and one of the most rapid forms of change.
A is too long-term; maybe the capacity to sweat a certain amount could be a genetic response, but the specific sweat response happens too quickly to be a genetic response.
Growth response is also wrong because growth responses are “relatively irreversible” according to paragraph 2. People who sweat don’t keep sweating for the rest of their lives, so B has to be wrong.
C matches my prediction, and the description of physiological responses in paragraph 2, so it is correct.
Though people sometimes describe sweating as a behavior, sweating is not a behavior in the way that the passage describes. In paragraph 3, behavioral responses are equated with “culturally patterned ways” of behaving. Since sweating is not a cultural practice, D can’t be correct.
This claim is first mentioned in the first paragraph: “the instability of vision that his pictorial strategy imposes on the viewer mimics the artist’s own bodily weakness.” So I’m looking for an answer choice that mentions something about unstable viewpoints.
The first paragraph mentions that Manet struggled to complete the project, not that he failed to do so, so A has to be incorrect.
Nothing in the passage suggests that Manet was painting loosely. The passage suggests the opposite, really: that a lot of thought went into the painting. So B has to be wrong.
C matches my prediction: “inconsistency” is a synonym of “instability,” and “viewpoint” is one of “vision.” That makes C correct.
The bar scene description comes later, as evidence of the second claim made in paragraph 1: that the perspectival system “forces the viewer to participate psychologically in the artist’s growing awareness of the tenuousness of the lifeline that bound him to his mortal existence.” Since D is mentioned after the author mentions Manet’s weakness, and as part of a new but related point, D can’t be correct.
This is a really challenging question: one that’s asking me to discern something on the basis of the description of the painting. Surprisingly, though, it helps that the passage does not spend much time describing the painting itself. I know from paragraph 2 that the waitress, Suzon, “stares blankly at the implied customer standing before her (who, although transparent to the viewer, appears enigmatically in the mirror behind her, an indistinct reflection, like the hereby crowd).” The last paragraph also tells me that Suzon is “a simple working girl, in her symmetrical, wrist-forward pose and central placement in the foreground of the composition.” Finally, I know that there is a “trapeze artist in the uppermost corner of the painting, indicated by only her ankles and feet.” Hopefully that is enough to find an answer.
The passage points to many aspects of the painting that reflect Manet: specifically Suzon, and perhaps the trapeze artist. That decreases the likelihood that Manet would need to put himself in the painting. Without more evidence to suggest it, I can eliminate A.
This choice confused me at first, until I remembered the mirror. There is a mirror behind Suzon, who is staring towards the viewer. I know the mirror is behind her because it reflects the customer in front of her that we cannot see. So that mirror would, I can deduce, likely also show Suzon’s head from behind. B has to be the correct answer.
I know that C is incorrect because the trapeze artist’s body—which is where the artist’s costume would be—is out of frame; she is “indicated only by her ankles and feet.”
This answer choice is interesting; if I was having trouble with this passage, D would actually help put me on the right track to figure out the right answer, since it draws my attention to the mirror behind Suzon. In any case, D has to be wrong because the mirror reflects Suzon’s front; the customer is in front of Suzon, somewhere behind us, facing her.
This is an uncommon question type: a paradox question. Some new piece of information—in this case, that the painting was painted well before Manet died—is introduced, which seems to challenge or contradict some part of the passage argument. I need to figure out a way to reconcile this new piece of information with the passage argument: to show that it is just a paradox, a seeming contradiction, instead of an actual one. The contradiction here appears to be that, on the one hand, the reading of the painting provided by the passage depends on Manet knowing that he’s about to die. But the information in the question stem says that he wasn’t about to die: that he would die twenty years later. The most immediate resolution that comes to mind is that perhaps Manet found out he was going to die for sure two decades earlier: back when he started to paint this painting. Most of us do not know when we will die; knowing exactly when would encourage us to come to terms with our deaths, as the passage argues Manet does.
A works as a correct answer. Knowing that he was going to die due to an illness, even if that death ended up coming much later, would change his mindset about the world.
This would not resolve the paradox. It might explain the subject of Manet’s painting, but not the approach and style Manet took, which the passage argues is inflected by the sense of his impending death. I can therefore eliminate B.
C doesn’t resolve the paradox either, since it does speak to the central problem of the paradox: why Manet is painting as if his death is impending, when his death is actually far off? C has to be wrong.
D is a great answer choice, but one that is plainly wrong. If D were true, that would make more problems for the author, not fewer, since it would invalidate every reference to Manet in the passage.
Resurrection is brought up in the very last paragraph: “Just as the Christian disciples watched their savior disappear from earthly view, we witness her escape from the confines of the framed composition. Like the Christ concept, Manet saw himself as destined to live on as a master. His corpus of paintings constituted his legacy to artists of the future and would consolidate his reputation as a pioneer of that new religious order, Modernism.” So the author brings up resurrection as a metaphor for Manet’s “legacy to artists of the future”: how his work would influence later artists even after his death.
This is an attractive answer choice because the trapeze artist is the prompt for this comment about resurrection. But the author claims that this is an “indirect reference”: in other words, a weak one. The author is much clearer with regards to how Suzon symbolizes Manet, so one can safely assume that if the author were to suggest that Manet also symbolized himself as the trapeze artist, the author would have been clearer about this. So A has to be wrong.
B matches my prediction—that Manet was concerned over his legacy—so I can choose it and move on.
At no point does the passage suggest that Manet hoped that his health would be restored. In fact, the passage reading depends on the idea that Manet had no hope that his health could be restored: that he accepted his death. So C has to be wrong.
D also has to be wrong because the author is pretty explicit about Manet’s beliefs in paragraph 3: “Whether Manet, who was ostensibly a Roman Catholic, believed in a personal immortality remains uncertain.” If it is unclear that Manet believed in eternal life, it is difficult to see the author’s comment about resurrection, which the author seems fairly sure of, to be a comment about Manet’s belief in an afterlife.
The question stem tells me that I need to figure out what effect the new information in the question stem has on the passage argument. That means I need to determine what part of the passage the new information affects. The nice thing about this kind of question type, though, is that I can eliminate at least two answer choices simply by determining if the information in the question stem would strengthen or weaken the passage argument. What the question stem really seems to describe is how other people were painting like Manet did in A Bar at the Folles-Bergère. That opens up the possibility that this passage, which is so focused on arguing that Manet painted this painting in a specific way in order to say something about his mortality, could actually have another cause: that perhaps Manet was just painting as others were painting during his time. So in other words, while this doesn’t disprove the passage argument—for that to be the case, I would have to have more information that clearly suggested that Manet was influenced by these contemporaries—it does weaken it, because it suggests that Manet could have had other reasons for painting this piece as he did.
A has to be wrong because the new information will weaken the passage, not strengthen it.
B is wrong for the same reason.
C is wrong because nothing in the passage depends on Manet being the only person interested in representing modern life. The passage instead depends on the assumption that Manet was depicting his private struggle with his own death.
D has to be right, and it matches my prediction: that the question stem opens up the possibility that Manet’s subject matter were not as private as the passage argues.
The right answer to this question will fit the definition of claim right in paragraph 6: “services such as education and public assistance provided in return for the performance of social duties.” So I’m looking for a right to some service: something done for a citizen who fulfills their obligations.
A works, since A is a service; emergency medical care is something someone could do for a citizen.
B is maybe the next most attractive answer choice here, but it is not as clearly a service as A is. A speedy trial is not necessarily something done for me—I participate in it, and that trial could end in my incarceration—while A is absolutely something that would be done for me. So B has to be a worse answer than A.
C is definitely wrong because it is also not a service. C allows me to behave in a certain way; it is not something that someone else can do for me.
D is similar to C insofar as it describes a behavior that I am allowed to do, and not a service done for me. So D is also wrong.
Political conflict is mentioned in the second to last paragraph as the first assumption that the French approach to rights makes: “politics is to be replaced by a science of society. The purpose of government is to reduce the space between what should be and what is. That space—its complexity, its injustice, and its inadequacy—is seen not as something inherent in any society but as the historical (and thus transcendable) result of a class struggle produced by an essentially elitist revolution. Claims by the groups constituting society thus arise from the very divisions and imperfections in the social order that government exists to reduce and ultimately to abolish.” The key here is that conflicts and disagreements occur in the space between what is and what should be that the government is meant to reduce. And that space is the result of history: of past “class struggle” that can be addressed and eliminated. So I’m looking for an answer choice that either mentions that political conflict can be eliminated, or that it is based on history.
olitical conflict does end up generating a demand for rights—when the passage says that “Claims by the groups constituting society thus arise from the very divisions and imperfections in the social order,” that means that the insistence on rights emerges because society needs to be better perfected—but that is not the same as protecting those rights. That alone makes A wrong. But A is also wrong because the author’s entire point is that the French system moves away from a model of rights based on individual liberties. That’s especially clear in the last paragraph, where the author states that “an insistence on individual liberties was no longer the solution; it was, in fact, the problem.” Those French political scientists would therefore not be interested in protecting individual liberties.
National leadership isn’t really discussed when political conflicts in France are discussed. Instead, the passage is focused on how political conflicts are how a society works out its historical imperfections. They are not, therefore, necessarily evidence of bad national leadership, so B has to be wrong.
Morality is not really discussed in terms of the French political system, so I can eliminate C.
D matches one of my predictions: that political conflicts are the result of historical problems. I can choose D and move on.
This question is asking for an answer choice which is not going to be confirmable: for which there is no procedure or piece of evidence that could fully prove the claim. I don’t even have to look at the passage in order to answer this question; I just need to evaluate each answer choice and ask if some imagined fact could confirm the answer choice.
I could prove A by providing examples from political science books or important documents in political history to show that the definition of liberalism has shifted towards an association with individual liberties. Since I can confirm A, A has to be wrong.
B is the correct answer here. Superiority is difficult to prove in general, without clear terms on which to judge whether something is superior to something else. Since no such criteria are provided, it is easy to imagine that a critical reader—someone who is intent on arguing with the passage—would not find any fact persuasive enough to convince them of B.
This could be confirmed by comparing the Constitution of 1848 with other documents that use rights in a way that gets equated with individual liberties. So C has to be incorrect.
And D can be proven with a survey of works on individual liberties that show people wondering if individual liberties are good or not.
Continental Europe is helpfully capitalized, which helps me see that it is in paragraph 4. There, the definition of liberalism is described as different from its meaning in the U.S. The specific examples from Continental Europe are Italy and Germany, “where a Liberal is now someone who supports the freedoms that favor the industrial and commercial elite.” So I’m looking for an answer choice which describes an action that supports the wealthy and powerful.
A is the opposite of what I’m looking for; such would threaten the power of the wealthy, not support them.
Paragraph 4 implies that there has been some divergence in the concept of individual liberties, but it does not imply that liberalism has come to oppose individual liberties. The actual quote suggests that liberalism has shifted its sense of whose rights are most protected: from individual people to wealth and powerful individuals. B is therefore wrong.
C is very similar to A insofar as it describes something which does not benefit the wealthy. So C has to be wrong as well.
Keeping workers from striking almost certainly helps the wealthy and powerful, since it keeps workers from gaining enough power to challenge their bosses. D is therefore correct.
To answer this question, I just need to have the rest of the quote from paragraph 5 in mind: “There, popular opposition to individual liberties that protected the privileged, a situation attributed to a liberal revolution, fused with social criticism of the early effects of industrialization to produce a new rights language.” The key part here is that idea that what people are opposed to are liberties that “protected the privileged.” That’s what the right answer has to mention.
A is correct. This is one way that rights have protected the privileged. The passage begins this claim by referencing what liberalism has come to mean in Italy and Germany in paragraph 4, and then builds into an argument in paragraph 7 about how the project of the French government is to eliminate social equalities that are partly in place because of the insistence on rights.
This is perhaps the most attractive answer choice. But the problem here is the same one that is brought up in Germany and Italy in paragraph 4: that individual liberties may protect individuals, but they protect some individuals more than others. So B is incorrect because it goes further than the passage does.
The passage actually suggests the opposite of C. To say that individual liberties protect the privileged means that a minority group—the privileged—is most protected. So C can’t be right.
The passage never says that individual liberties are impractical. All that paragraph 5 claims is that the French saw individual liberties as functioning in a way that people disagreed with. So D has to be wrong.
The question stem provides a claim from paragraph 4. It is part of a paragraph that argues that an idealized account of liberalism developed into divergent ideas of the concept. Most specifically, the claim in the question stem follows another point: “its meaning in Britain and the United States diverged radically from its meaning in Continental Europe.” So it is very likely that the claim about what liberal means in America is meant to help illustrate this divergence.
A matches my prediction, and so I can choose it and move on.
Paragraph 4 actually argues that liberalism moved away from this idealized definition of the concept. So I can eliminate B.
C has a similar problem to B. Both describe developments in the concept of liberalism and individual rights that precede, historically, the description of liberalism and conservatism in U.S. politics. So C has to be wrong as well.
France isn’t mentioned until paragraph 5, so while D is implied by the passage, it is not what the description of U.S. politics is meant to serve as evidence for. D is therefore wrong.
I can tell what an example is meant to argue by just noting the argument closest to it. Or, I can discern the intent of an example by what it actually demonstrates. Paragraph 2 begins with an argument: “Although art has always been a commodity, it loses its inherent value and its social use when it is treated only as such. To lock it into a market circus is to lock people out of contemplating it.” So I can guess that the right answer has something to do with the damage done when art becomes a commodity. And if I look at the example itself—“If there were only one copy of each book in the world, fought over by multimillionaires and investment trusts and then hidden in storage, what would happen to one’s sense of literature”—I get a similar sense: that it becomes difficult to gain a sense of art when it is locked away.
A is partly true, but it lacks a key aspect of the argument in paragraph 2: what negative effects occur because art is expensive. This is because A is actually established in the first paragraph, while paragraph 2 builds on that argument. So A is incorrect.
There is no clear discussion of how things have changed over time in paragraph 2, so I can safely eliminate B.
New York is not mentioned in paragraph 2 either, so the example is unlikely to argue for C.
D is the last answer, but it is definitely correct. It matches my prediction: that there is something damaging about limiting access to art when art becomes a commodity.
This is a direct reference to the last paragraph: “American museums have in fact been hit by a double whammy: art inflation and a punitive rewriting, in 1986, of U.S. tax laws, which destroyed most incentives for the rich to give art away.” If the wealthy began to donate even more art to museums, then it is unlikely that the government actually did anything to hurt museums.
A matches my prediction. So A is correct.
B is wrong because the last paragraph states that B is the first “whammy” of the two that have hurt American museums. The credit economy point is not directly related to the arguments about donating to art museums.
C has a similar problem to B: it relates to the commodification of the art market, and not the tax laws which reduced incentives to donate music to art.
D runs into the same problem as A and C. The buying power of museums does not have anything to do with the rates at which the wealthy donate to museums because it is associated with a different point: how tax laws have made the rich less likely to donate to museums.
My initial reading of this passage tells me that there is a list of negative effects in paragraphs 3 and 4. Paragraph 3 states that “It has distorted the ground of people’s reaction to painting and sculpture” by making people think of price whenever they are trying to just appreciate art. Paragraph 4 claims that museums can no longer afford to buy art because art has become too expensive.I. distorting viewers’ responses to paintings and sculptures. I is correct because it is a direct quote from paragraph 3.

II. depreciating the value of gifts previously given to museums. The passage does not really mention depreciation of art. It only talks about how art keeps increasing in value. So II is incorrect.

III. Diminishing the buying power of art museums. III is mentioned in paragraph 4, so it must be correct.

II is not mentioned in the passage, so A is incorrect.
B is wrong because II is incorrect.
C matches my prediction, so C is correct.
And D has to be wrong again because II is wrong.
This once again refers to paragraph 4: “there are many areas in which American museums can no longer buy […] as their once adequate annual buying budgets of two to five million dollars are turned to chicken feed by art inflation. And as the museums’ buying power fades, the public experience of art is impoverished.” So I’m looking for an argument that argues that museums can no longer afford to buy art.
A is meant to be tempting because it sounds like my prediction, but it’s actually a distortion of the passage. Paragraph 4 doesn’t argue that museums have lost their budgets: that they don’t have money. The passage only argues that the money they do have cannot keep up with the increases in the price of art.
Insurance is not mentioned as a cost in the passage, so B is incorrect.
There’s no reason why museums would have to reduce their viewing hours, so C is incorrect.
D matches my prediction, so I can choose it and move on.
This is a general weakener, so I’m looking for a choice that attacks the main idea: that the commodification of art has hurt art. The most immediate ways to attack this claim are either to suggest that art has not been hurt by rising art prices, or that art prices haven’t been rising.
A does not match my prediction, but it very directly attacks a claim made in paragraph 3: that increases in the price of art “has distorted the ground of people’s reaction to painting and sculpture” by making viewers always wonder about the price of art at auction. This is a key part of the author’s argument: that the cost of art negatively affects art itself. This attack means that A weakens the argument, and so must be correct.
B actually would confirm the author’s argument, since museums having to raise funds to buy art would suggest that art museums definitely do not have budgets big enough to buy as much art as they should (paragraph 4).
This is an attractive answer choice if you assume that a thousand paintings is not many, since that might imply that the art market is perhaps not being so commodified. But ten million dollars is a lot of money, and even if C were true, it leaves open the possibility that the vast majority of art sells for 8 or 9 million dollars. In other words, C is not a direct enough attack on the passage argument to make it correct.
D is meant to work in a similar way to C, insofar as it is meant to make me think that the art market might be cooling down if a nineteenth-century painting went for less money. But I have no idea what painting D is referring to, and so I don’t have the context to know if this one painting could be indicative of changing trends in the art market overall. Since D is not directly related enough to the passage argument, I need to eliminate it.
This is a main idea question: one that is asking me to confirm that I understand the moral system described in the passage. The heart of the passage focuses on how, for the Akan, morality is based on the actions that one does. I also know that the Akan are born morally neutral and so becomes better or worse by doing good or bad actions, and that one’s propensity to do good of evil can be affected by one’s sunsum, or one’s spirit. That should be enough for me to test each answer choice.
A goes against the passage discussion, since while people are born morally neutral, “The person’s original moral neutrality later comes to be affected by actions, habits, responses to moral instruction, and so on” (paragraph 3). The passage states many times that people can become virtuous by virtue of their actions, so A is wrong.
B also goes against the passage. People are born morally neutral, not virtuous, so B is already incorrect. The passage also states that unvirtuous people can perform virtuous acts, so B is doubly wrong.
C goes against the last paragraph, which states that “sunsum can be developed,” so even if you have weak sunsum, you can learn behaviors that strengthen that sunsum and help you to be more virtuous. C is therefore incorrect.
D is the last choice standing, and it is clearly correct. It specifically references the question regarding how someone who is morally neutral at birth learns to be morally good: “The answer is through moral instruction, which in traditional Akan society was normally done by means of ethical proverbs and folktales.” Since D corresponds with the passage discussion of Akan morality, D is correct.
The author brings up the same problem with Akan moral philosophy in paragraph 4—“There is one difficult question inherent in the Akan position, a question that has not been squarely faced or examined by scholars. The question is this: How are we to perform virtuous acts if we are not already virtuous?” But the next few paragraphs directly answer this: by using moral instruction. The other problem with this philosophy that the passage raises is mentioned in the last paragraph: “the Akan thinker’s position here cannot claim to be final and impregnable, for we can still pose the questions: Is it not the case that making an effort itself requires or presupposes antecedent factors? If so, what are these factors and how can they be acquired? The attempt to answer this set of questions will most probably involve us in a circle, and a vicious one.” So I’m looking for an answer choice which describes the problem of what encourages someone to act morally: whether there needs to be something inherent in the person to encourage that behavior.
The author never contests the value of moral instruction, so I can eliminate A.
This is never raised as a problem in the passage. People are morally neutral at birth; instruction doesn’t affect that fact, it just helps cultivate their morality after birth.
C is the correct answer. The author’s two objections are actually the same objections: that there may be something in a person that needs to preexist in order to make that person act morally, or be more receptive to moral instruction than others, or be more willing to strengthen their sunsum than others.
The author does not contest the idea that people can change their character. That part of the Akan philosophy seems acceptable to the author. So D is incorrect.
The Akan words for goodness are translated in paragraph 2: “The last syllable of each word means to do or perform. Thus, the two words literally mean ‘good-doing’ (that is, doing good).” This is used to help reinforce the claim from paragraph 1: that “It is on the basis of a person’s conduct […] that the Akans […] judge one to be good or bad.” So the answer must be that the Akan words for “goodness” are there as proof that the Akans believed that actions determined one’s moral standing.
A is incorrect because this point is made much later in the passage, as a concern raised by the author: not the Akans.
B is incorrect because the passage claims that morality is generally concerned with good or bad actions. That’s why the word “doing” is so emphasized by the author in paragraph 2.
C matches my prediction, and mentions actions, so C is correct.
This claim, like A, comes later in the passage, and is not clearly related to the Akans using a word that includes “doing” in the passage.
My work on prior questions helps me with this one. I know that the problem in the final paragraph is whether or not “making an effort” to improve one’s sunsum “requires or presuppose antecedent factors.” So I’m looking for an answer choice which suggests that there is no such need: that addresses the author’s concern that there might need to be something virtuous in people to help them become virtuous.
A works. If everyone had an equal ability to exert effort to improve, then one would become good or bad by virtue of one’s actions: in this case, by trying to be good. I would choose A and move on.
B would not affect the author’s primary concern: that the Akan don’t adequately explain why some people put in effort to improve their moral behaviors, and why some do not. This is because B doesn’t deal with the problem of different quantities of effort; that makes B wrong.
C is incorrect because the author accepts C, and still has a problem with it. What, the author asks, dictates whether someone expends the effort to strengthen their sunsum or not? Since C doesn’t add anything to the passage argument, it can’t resolve the author’s concern.
D also fails to touch on the author’s concern. The author has no problem with D, but would still ask: what makes moral instruction more effective in some people and not in others? Since D has no answer to this problem, D can’t be right.
This is stated in paragraph 3: “The Akan position is that the original nature of human beings was morally neutral. If this were not the case, there would be no such thing as am oral person.” So The only reason people can be considered moral or immoral is because they are born morally neutral.
This might account why people are more or less moral, but it doesn’t explain why people can be considered moral at all. A is therefore incorrect.
This matches my prediction, so I can choose it confidently.
Moral education helps people to be moral, but it does not affect their ability to be considered moral or immoral. That’s a more fundamental problem that instruction does not affect. C is therefore wrong.
I can’t pick D because it’s not in the passage – it was an answer choice 2 questions back! So it must be wrong.
This is an assumption question, so I’m being asked for some claim or idea that the passage does not state, but that is necessary to believe for the passage argument to make sense. This question is specifically pointing me to the last paragraph, where the passage author brings up concerns about what makes someone expend effort to improve their moral behavior: “Is it not the case that making an effort itself requires or presupposes antecedent factors?” That seems to assume that people need to be stimulated to expend effort: that they will not just naturally put in the effort to morally improve without some prompting. Note that this is not explicitly stated, but it is clearly implied by the passage argument, so it must be what the right answer will say.
The author’s concern over Akan philosophy is not based on the claim that all philosophies have flaws. The author is instead making a very specific claim: that Akan moral philosophy may not fully account for how people expend effort to morally improve.
Western thought is not mentioned at all, so there’s no way that the author needs to assume it.
The author actually assumes the opposite: that effort is not self-motivating, that something needs to motivate effort. If the author believed C, then it would not make sense for the author to ask from where the effort to change oneself comes from.
D matches my prediction, and so I can choose it and move on.
The key reference here is to “moral instruction.” That is mentioned in paragraph 4: that it is through moral instruction that “the growing child and young adult become aware of what is a virtuous or vicious act and become virtuous by performing virtuous acts.” That should be what the right answer says.
The information in the question stem seems to refer to someone who is morally bad at first and then becomes morally good, so I’m not sure how the situation in the question stem would suggest that people are born morally neutral.
B doesn’t exactly match my prediction, but it has to be right. Moral instruction, paragraph 7 suggests, can develop one’s sunsum: one’s spirit, what pushes one to act morally or immorally.
The situation in the question stem doesn’t mention that the person did virtuous things. It only mentions that they did something evil. So C isn’t supported by that situation.
This is incorrect because the situation in the question stem talks about moral instruction, so the right answer has to say something about the change from being a bad person to being a good one. Since D does not, D is incorrect.
This question tests how well I understand Akan philosophy. A bad person is someone who does bad things, but the real question here is: why can they be blamed for doing bad things? Given that the passage mentions moral instruction, and how people with weak sunsum can strengthen their sunsum, the underlying reason why people can be blamed for their actions is that they could do better: that there are mechanisms and systems in place through which they could behave more morally.
A matches my prediction—that everyone can do better—so I can choose it confidently.
B would actually make someone less culpable for their actions. Moral instruction is how the Akan claim people learn what a virtuous or a vicious act is (paragraph 5). If an Akan lacks this instruction, then they can’t know what a good or bad action is, and if someone doesn’t know what good or bad actions are, then they can’t be blamed for their actions.
C is incorrect, because the last paragraph claims that even people with weak sunsum can develop that sunsum and strengthen it. So even if I have weak sunsum, which might make me more likely to do bad things, I could always develop that sunsum and so become a better person. That means that it is not the sunsum itself that is to blame, but rather my willingness to develop my sunsum.
Suban is mentioned in paragraph 6 as a word for “character,” and I’m already inclined to eliminate it because it plays such a small role in the passage. But D has to be wrong because it would not make sense to say that people can be blamed for having bad character because they have bad character. Since that’s just logically incorrect, D has to be wrong.
The quote from the passage clearly needs to explain what the status quo is in order for the passage to claim that the status quo is a powerful force. That explanation can occur before or after the quote, so it pays to look at both. The sentence before states: “Tom Lindsley, of the National Alliance of Business, says that executives find it just too daunting to have to lay the groundwork of reading, writing, and arithmetic before they can even begin to teach employees new skills that arise from the latest technology or work reorganization.” So I know that the status quo has something to do with how challenging executives find training workers to be. The sentence after the quote reads: “most companies concentrate on the short term. It’s a focus encouraged by Wall Street, as well as government policies and by tax laws, that—for instance—allow tax breaks for a fancy new building but not for teaching employees new skills that would allow the company to become more competitive.” That also talks about how hard it is to train new employees, though it is a bit more specific as to why: because it pays to focus on the short term, which is better subsidized by the government. So I’m looking for an answer that either talks about how hard it is to train employees, or that mentions how much short-term thinking is encouraged.
Paragraph 5 doesn’t bring up worker productivity, so I can safely eliminate A.
B matches one of my predictions, since it implies that it is difficult to train employees. I would choose B and move on.
This claim is made in paragraph 6, which makes a different point than paragraph 5, so C can’t be right.
D is probably meant to be a pretty attractive answer choice, because tax breaks are mentioned in paragraph 5. But D distorts the passage argument; paragraph 5 says that the status quo is “encouraged by Wall Street, as well as by government policies and tax laws.” For that to make sense, those tax breaks can’t be a part of the status quo. That makes D incorrect.
My work on the previous question should help me out with figuring out what the passage assumes: that is, a claim that the passage necessarily holds, but that goes unstated. I know that the author brings up tax laws in order to help argue that executives are disincentivized from planning for the future. The specific claim about tax breaks, though describes how there are “tax breaks for a fancy new building but not for teaching employees new skills that would allow the company to become more productive.” The phrasing here implies that there is an opposition between the “fancy new building” and “teaching employees skills that would help the company to become more productive,” and an opposition implies that the opposites are unlike each other. So I can deduce that the passage assumes that fancy new buildings won’t help the company to become more productive, since the description of productivity is associated with teaching employees skills. I know that has to be the assumption because the passage does not explicitly state this, but if I don’t believe this, then the argument does not make sense.
Nothing in the description of the tax breaks seems to require that employees need to know about those tax breaks. The paragraph actually talks about the decisions that executives make, and not workers, so A can’t be right.
Paragraph 5 does not oppose the educational system and businesses. The passage is ultimately about what form tax breaks for businesses should take: employee training or new buildings. B therefore is not clearly assumed by the argument about tax breaks, and so is incorrect.
The passage actually implies the opposite. Paragraph 5 states that “most companies concentrate on the short term” before bringing up the example of the tax breaks, which implies that the tax break is an example of short-term thinking. And if the building is an example of short-term thinking, it is unlikely to involve long-term commitments. C is therefore incorrect.
D matches my prediction, so I can choose it and move on.
High-performance workplaces are discussed in paragraph 3. Its attributes include people working “in teams” who are “cross-trained, lightly supervised, and empowered to make decisions that used to be reserved for the bosses.” The paragraph also mentions that these have state-of-the-art technology, and “employ fewer people than they did before they were reorganized.” Since the question is asking me about a potential weakness to this approach, the feature that stands out to me is the last one. While there may be benefits to employing fewer people, there are also downsides: having less manpower, and of course having more people unemployed. So I’m going into the answer choices predicting that as my correct answer.
The passage states the opposite: that employees will be “cross-trained,” and so will not be too highly specialized.
I don’t see anything about tax breaks and how having a high-performance workplace might affect them, so B has to be wrong.
The discussion of high-performance workplaces only focuses on skilled workers, and does not discuss unskilled workers, so C has to be wrong.
D, though, matches my prediction. It makes sense that requiring fewer employees would also increase unemployment to some extent. Since D is grounded in the passage discussion of a high-performance workplace, D has to be correct.
The first paragraph stood out to me in my initial reading of the passage because it brought up the national implications of the problems faced by U.S. manufacturers: “A declining standard of living for the entire country as well as a crop of social problems brought on by the inability of a large, fast-growing segment of society to make a decent wage.” That means that the author would likely want high-performance workplaces, which are described as a possible solution to the problems faced by U.S. manufacturers, to be implemented nationally, since national problems intuitively need national solutions.
A matches my prediction. It fits a passage that is ultimately trying to come up with a solution to a problem faced by U.S. manufacturers, so I can choose it and move on.
Even though international business is discussed in the passage—insofar as U.S. manufacturers have to compete with products coming from international manufacturers—the passage is focused on what U.S. manufacturers need. Since a high-performance workplace is likely to give an advantage to whoever uses them, and since that concern over competing with international manufacturers implies that the author is concerned with U.S. competitiveness, it is reasonable to assume that the author would primarily want high-performance workplaces to be a national solution—an American national solution—and not an international one.
There is no clear indication in the passage regarding how quickly the author wants high-performance workplaces to be implemented. But the seriousness of the problem the passage is trying to solve at least makes it possible that the author would want high-performance workplaces to come sooner rather than later. So C has to be wrong.
The passage is primarily focused on the manufacturing sector. Paragraph 1 clearly distinguishes between the manufacturing sector and the service sector when it says that “Modestly skilled workers can either hold onto those bargain-basement jobs or bolt to the service sector.” Since D does not describe manufacturing, there is not enough in the passage to justify choosing D.
I know that paragraph 2 mentions that “Everybody says better trained workers would make the nation competitive again.” The next sentence begins with “Hence” and then mentions the federal Commission on the Skills of the American Workforce, which implies that that commission must agree with that statement.I. Since I is mentioned directly after the statement about what “everybody says,” I has to be correct.

II. Workers are not explicitly mentioned in either paragraph 2 or 3 as agreeing with what “everybody says,” so II is incorrect.

III. I know that management theorists are mentioned in paragraph 3, where they are used to explain the first sentence of that paragraph: “What is this ‘high-performance workplace’ everybody says is needed?” Since they explain what high-performance workplaces require in a way the passage describes as “faithful,” they must agree with what “everybody says is needed,” so III is also correct.

II is the only incorrect option, so A is wrong.
III is correct, but so is I.
C contains both I and III, so it is correct.
D contains II, so it is incorrect.
To answer this question, I need to discern who the passage arguments benefit. I know that the wrong answers will describe people who would benefit from the passage argument, and the right answer will describe someone who would not benefit: either because they are excluded from the argument, or because the passage argument somehow argues against them. That said, because the answer choices could describe anyone at all, it is difficult to make a more specific prediction than that. So I should just move to the answer choices and ask how a passage arguing for high-performance workplaces—and explaining why those workplaces are difficult to create—would affect the people described.
I know from my work on previous questions that the passage is more focused on what employers can do than it is on what workers can do. That all suggests that unemployed people who are not highly trained might not get much out of this passage, since they cannot train in the ways that they might need to find work in high-performance workplaces. That makes A correct.
The passage calls out ways that government policies get in the way of U.S. manufacturers switching to high-performance workplaces in paragraph 5—where the passage talks about tax breaks for buildings and not for worker training—so I can imagine the passage could benefit government officials by encouraging them to use policy to encourage employee training. That makes B wrong.
Manufacturing sector corporate managers would absolutely benefit from the passage, since the passage is all about problems that their sector faces. So I can eliminate C.
The passage mentions students of management and is making a point about management changes, so it’s not hard to imagine that students of management could learn something from the passage discussion. D is therefore incorrect.
The passage is full of reasons why income inequality is getting worse, but it never says that one reason is more likely to worsen the trend toward economic inequality than another. That means that the right answer will be the only answer mentioned as a reason for income inequality in the passage. Those reasons are: education (paragraph 2), baby boomers crowding the market (paragraph 3), multiple incomes (paragraph 5), assortative mating (paragraph 6), and wealth distribution (last paragraph). I can also attack this question by eliminating the wrong answers, which will either suggest the opposite of these reasons, or say something random.
Paragraph 2 suggests the opposite of A: that an increase in the number of assembly-line jobs that pay well would help address the problem of income inequality caused by differences in education. So A has to be wrong.
B resembles paragraph 3: “The advent of the boomers produced a natural bulge at the lower end of the wage scale. Worse, as these boomers competed with each other for entry-level work, they bid down entry-level wages—a phenomenon called ‘crowding.’ Between 1973 and 1987, the real (adjusted for inflation) incomes of full-time younger workers fell, while the incomes of older workers rose. Unskilled younger workers took an especially big hit.” It stands to reason that if it is bad to have too many people competing for entry-level work, then even more young workers entering the job market would exacerbate this problem and lead to more income inequality. B therefore has to be correct.
C would arguably allow more people to go to college, which would then minimize the effect that unequal access to higher education has on income inequality. C therefore has to be incorrect.
D would directly help the poor and hurt the wealthy: thus redistributing wealth in a way that helped reduce, not increase, income inequality. D thus has to be wrong.
This sounds like a flaw question—a question asking me to determine an error that the author makes while arguing—but the author brings up problems with the demographic explanation in paragraph 4: “The demographic explanation is supposed to be reassuring, on the theory that income differences caused by the baby boom will cure themselves over time. The only problem is that it isn’t happening. Boomers started hitting their mid-thirties in 1980.” So the problem with the demographic explanation is that it should be the case that boomers would make more as they grew older, and should thus lessen income inequality. But that isn’t happening. So the answer has to mention something about how boomers aren’t making more even as they grow older.
The passage doesn’t bring up boomers graduating from college. Education is a separate cause of income inequality, so there’s no reason why the author would risk confusing readers by conflating education and the demographic explanation. A has to be wrong.
This point comes later, and is therefore wrong for the same reason A is: because B is a distinct point, and the author would not want to risk confusion by mixing the demographic argument and the combined income argument.
C matches my prediction: that, as paragraph 4 says, the demographic explanation fails to account for what happened to boomers once they leave entry-level jobs.
D is a more detailed version of C, but that makes D worse because these details aren’t mentioned in the passage. So I can eliminate D because the description of boomers doesn’t include anything about purchasing consumer goods.
This is another quick passage check. Education is discussed in paragraph 2, where the author begins by saying that in the period described, “the economic advantage of a college degree actually diminished.” However, since the range also mentions the mid-1980s—an inexact number—I should also include what happened in 1986: “then the advantage of the college educated grew rapidly until, by 1986, the gap in earnings had widened to 49 percent.” So the right answer will likely mention something about a decline in college graduates’ earnings advantage, and then a rapid rise.
Nope. The right answer definitely has to mention the decline, because that is key to the author’s argument, so A is wrong.
No decline here, so B is also wrong.
C matches my prediction, so I can choose it confidently.
This choice is meant to be attractive because it uses a term from the passage: “real.” When an answer choice uses terminology from the passage, that feels reassuring. But that terminology comes up in paragraph 3, when the author brings up baby boomers. That’s a different point than the author’s discussion of the role of education on income inequality, so D has to be wrong.
Assortative mating is discussed in paragraph 6, where the author describes how people who marry those with similar incomes end up increasing inequality. If it is the case that women’s lower wages have masked this effect, then trying to even out the gender gap in pay would make the income inequality caused by assortative mating even worse. That feels pretty good for a prediction.
A matches my prediction perfectly. The author implies at the end of paragraph 6 that the lower wages of women makes heterosexual couples’ incomes look lower on average. So raising the wages of women to match those of men will mean that the average income of these assortatively mated heterosexual couples will go up, which will in turn increase income inequality. I can choose A and move on.
B is immediately wrong to me because it talks about “randomly mated heterosexual” couples. I don’t know anything about randomness based on the passage, so I can eliminate B.
C has a similar problem to B, so it’s also wrong.
The passage implies the opposite: that since gender discrimination means that the women in heterosexual couples earns less, that the effects of assortative mating are lessened by gender discrimination, because a man marrying a woman is akin to a high earner marrying a lower earner (instead of marrying someone who earns a similar amount). D therefore has to be wrong.
This question is asking me to find a passage claim which supports a point that is not made in the passage. I know that education is only discussed in paragraph 2, so the answer could be there. Or the answer could be in paragraph 3, which talks about a diminishing pool of jobs due to “crowding.” I should be prepared for either paragraph to be invoked.
The question stem is asking about problems with American education. The passage limits this discussion to higher education. So non-college-educated men would not be indicative of failings by the American education system. A therefore has to be wrong.
I can eliminate B because it implies something positive about the American education system, while I’m looking for something which indicates a problem with that system.
I have no idea how C relates to the education system; I’d need more data regarding what share of the nation’s college graduates joined the lower class in the time period described before I could justify C. That means that C has to be incorrect.
D is the last answer, so it has to be correct. It also matches my second prediction by referring to paragraph 3 and the diminished pool of jobs. A smaller pool of jobs—that is to say, a smaller number relative to the number of people seeking those jobs—means that wages could shrink. This could be a problem with the American education system because that system failed to train the younger workers of that generation to take entry-level jobs that older workers with more training would have squeezed them out of.
The author spends the first few paragraph explaining why doctors think that lying to patients is justified, so part of the answer has to be that the author understands why physicians do this. But the second half of the passage condemns the practice, so the right answer has to mention that too.
A has to be wrong because it does not account for the first half of the passage. The author is too understanding of the physician position to show contempt for this.
B matches my prediction. The disapproval comes out in paragraph 4-6, while the comprehension comes out in paragraph 1-3. I can choose B and move on.
The author never seems confused by the decisions that doctor’s make, and the author is certainly not tolerant of this practice by the end of the passage. So C has to be wrong.
The passage arguably provides justification for why physicians do what they do, and maybe I can say that there is some regret when the passage author justifies the act of lying to patients, but D has to be wrong because the right answer also needs to mention how the author condemns the practice of lying to patients.
This is a straightforward passage check question. I could devote some time trying to predict what the author would definitely not argue, but more often than not, that would take awhile to do. And while the right answer could be something that the author opposes, it could also be a random answer choice, which would mean the time I spent trying to predict the right answer would be wasted effort. So the safe strategy here is just to check each answer choice and see what is argued by the author.
Paragraph 3 makes this claim. When the author says that physicians “may prescribe innumerable placebos,” they mean that they could prescribe treatments that are not medically justified. A is therefore incorrect.
This is part of the author’s arguments regarding why physicians should tell patients the truth. So B has to be wrong.
Here’s a good example of why it’s important to check the passage on a question like this. This argument is mentioned in the passage, but it is not a claim that the author argues. Instead, the author brings this up as an argument made by some physicians trying to justify lying to patients in paragraph 3. So C is the correct answer.
The author makes this point when discussing why doctors should not lie to patients, so D is also incorrect.
I know that the author is generally against doctors lying to patients. But the question stem’s reference to gravely ill patients brings to mind some of the justifications for lying to patients in paragraph 3: “Studies show that most doctors sincerely believe that the seriously ill do not want to know the truth about their condition, and that informing them risks destroying their hope, so they may recover more slowly, or deteriorate faster, perhaps even commit suicide.” If any of these were shown to be true, that would weaken the author’s claim that doctors should not lie to patients.
A matches my prediction. It is the obverse of what physicians are worried about; for the passage to cite doctors who believe that if people think their conditions are serious, they will deteriorate more quickly, those doctors also have to believe that if people think their conditions are less serious, they will heal better and not deteriorate as quickly. I can choose A and move on.
B wouldn’t clearly affect the author’s position. The author might even agree with B, and point out that that’s part of the problem with lying to patients.
The author would definitely agree with C. The passage mentions as much in paragraph 5, which describes how lying to patients gives them a false sense of hope, which could affect what decisions they make.
D is meant to sound like the part of paragraph 3 that the right answer is based on, but don’t know what being less cheerful has to do with being more or less healthy. I need an argument that the passage does not make—that less cheerful patients are less healthy—to justify D. The passage only says that telling patients that they are at risk destroys their hope, which is not quite the same as an emotional disposition. D therefore has to be incorrect.
Given that the passage argues that it is wrong to lie to patients, it is reasonable to believe that the situation in the question stem speaks to the problem of lying to patients. But medal malpractice is specifically referenced in the last paragraph: “The suspicion of deceit undercuts the work of the many doctors who are scrupulously honest with their patients; it contributes to the spiral of litigation and of ‘defensive medicine,’ and thus it injures, in turn, the entire medical profession.” So I’m looking for an answer choice which mentions that the rise in medical malpractice suits decreases trust in doctors more generally.
A is incorrect because there is no necessary relationship between medical malpractice and doctors caring about their patients. Patients can sue doctors that care for them, after all.
I don’t see anything in the question stem about patients having to cope with hospitalization; B would only weaken the argument if it would more clearly blame doctors for these cases of medical malpractice.
This is probably the most tempting answer choice, since it seems most in line with the main idea: that doctors may do some damage when they lie to patients. But C has to be incorrect because “do no harm” is not a legal requirement of physicians; it is instead a philosophy. So malpractice lawsuits can’t necessarily suggest that doctors are harming patients.
D matches my prediction: that the actions of doctors today (that is, their tendency to lie to patients) somehow hurts the profession by encouraging people to mistrust doctors. D has to be correct.
I know the author’s main idea: that doctors should not lie to patients. So I need an answer choice which discourages doctors from lying, or that speaks to the reasons why the author does not want doctors to lie to patients: because telling patients the truth can help them with their illnesses, and because “they invade the autonomy of patients and render them unable to make informed choices concerning their own health” (paragraph 5).
A is a bit too broad to be correct. The author does not suggest that doctors should not be trusted; rather, the author wants doctors to behave differently.
B feels like a pretty good answer choice, since it speaks to a reason why doctors should not lie: because doing so violates the autonomy of patients, which is rooted in the fact that doctors are treating their bodies. Still, I’m not absolutely sure about B, so I’ll hold onto it and eliminate the other answer choices before choosing B.
The passage does not suggest that only the helpless should not be lied to; the author does not want doctors to lie to any patients. So C has to be incorrect.
D is also too narrow; the author’s point about doctors concerns all patients, not just those who are in danger of dying. D therefore has to be wrong.
The idea that people at different levels of education experience literature differently is mentioned in paragraph 4: “over the last hundred years, the major canonical works have defined the syllabus at the higher levels of the system, while at the lower levels, the canon itself has been gradually replaced by a range of children’s and adolescents’ works useful for disseminating basic literacy because of their relative verbal simplicity.” This point is immediately elaborated on with examples of such works. So the right answer has to mention works written for children and adolescents.
A matches my prediction, so it must be correct.
The number of canonical women writers is mentioned much earlier in the passage to support a different point: that “the social stratification of speech has corresponded roughly with the level of class and less with gender” (paragraph 2). That makes B incorrect.
C doesn’t clearly have anything to do with the idea of lower and higher levels of education, so I can eliminate C.
I don’t see any such works mentioned when the author makes the point about lower and higher education and their linguistic goals, so D too has to be incorrect.
Paragraph 4 helps me here too. When the author claims that “over the last hundred years, the major canonical works have defined the syllabus at the higher levels of the system, while at the lower levels, the canon itself has been replaced by a range of children’s and adolescents’ works,” the implication there is that the canon is a group of texts.
The passage argues that studying the canon may encourage a sophisticated understanding of language, but the passage never states that the canon itself is that understanding of language. I can therefore eliminate A.
B is incorrect because it is a bit too similar to A. Both equate the canon with understandings of language, as opposed to the texts that impart that understanding of language to us.
C matches my prediction, so I can choose it and move on.
D is wrong for the same reasons that A and B are: because it suggests that the canon is what the canon helps us learn.
Since the question stem is asking about general inferences, it’s not really worth my time to predict what the right options will look like. I’m better off just evaluating each of the options.I. I sounds correct. If the books taught in public schools are chosen because they can teach basic literacy, then it makes sense that changing those books would change how literate public school students are, and with them, the population as a whole.

II. II is incorrect. I know from paragraph 3 that older works provide more outdated models of grammar, so a student who reads older works risks being less literate, not more.

III. III fits the concern mentioned in the question stem. The author’s main idea is that the canon and works of true literary merit are often locked away from people of lower classes, who get taught books which teach them grammar, but lack literary richness. So III has to be correct.

II is an incorrect option, so A is wrong.
I is also a good option, so B is wrong.
II is wrong, so C is wrong.
D has both I and III, so it is correct.
Another question stem which I can only address by testing each answer choice to see which answer is not supported by the passage. That means the right answer will either be something that the passage author disagrees with, or else it could be something random.
A is correct. The passage never says anything about who determines what makes it into the canon.
This is mentioned in paragraph 4, so I can eliminate B.
This is an effect of the author’s main idea: that your access to literature, which is determined by what class you are a part of, affects your speech. So C is also wrong.
This point is made in paragraph 5, and is also a clear suggestion of the author’s main idea, so D has to be incorrect.
Though this question refers to the first paragraph, it is really a main idea question. The author explains that part of what informs the formation of the canon is that “the language of societies with written literatures has thus tended to become internally stratified according to the groups among the population that have access to the school and the amount of access each has.” So the social order is served by separating the upper and lower classes, and by making sure that people from different classes learn different things.
A matches my prediction, since it is another way of saying that people of different classes learn different things. I can choose A and move on.
The first paragraph describes the relationship between literature and language: “Literature and language have marched through history in tandem with one another, and yesterday’s literature has become today’s grammar.” But nothing in that description mentions the need to obscure the relationship between the two. B therefore diverges enough from the passage to be incorrect.
The passage mentions giving the underprivileged literacy, but it also suggests that the wealthy become more literate than the poor. Since canon formation plays a role in this, C can’t be correct.
I don’t know anything about the class of the writers that are and are not chosen by the canon; the passage mainly discusses the class of students and readers. That makes D incorrect.
This is a straightforward strengthener, so I’m just looking for an answer choice which mentions how the canon is built in a way that stratifies literature and linguistic learning.
I’m not sure how A could strengthen the argument. It might suggest that more people are buying a book that paragraph 4 claims helps people to learn basic literacy, but I’m not sure what that says about the canon and social stratification. Since A’s impact on the argument is not very clear, I can eliminate it.
B runs into a similar problem to A. The answer choice wants me to think that only the poor masses are watching this show, but it is possible that this television program is actually watched by the upper and lower classes. Since B doesn’t reference class enough, I can’t be sure that it strengthens the passage argument.
C mentions class, but it’s too broad in scope to be correct. C refers to all advanced degrees, while the passage is primarily focused on literature.
D has all the elements that it needs to be correct. It mentions class, and how the richness of a piece of literature is only accessible to people of a certain class. I can choose D and move on.
As with most LEAST questions, it is often better to eliminate the wrong answers than it is to try and predict what the right one will look like. This is because I know what the wrong answers will look like: they will be instances in which gossip is likely to dominate conversation. To figure this out, I can go to paragraph 3, where gossip is defined as “informal personal communication about other people who are absent or treated as absent. It lacks formal rules setting forth who may speak and in what manner and with what limitations from the point of view of accuracy and reliability. It takes place spontaneously and relies on humor and guesswork, and it is casual with respect to who ends up receiving the information, in spite of the frequent promises not to repeat it that are ritualistically exacted along its path.” What seems to bind these different aspects of gossip together is informality: that gossip emerges in settings that are not strict or formal. That actually might be a good prediction for the right answer.
A has some aspects going for it. This committee is operating professionally, and so this situation might be too formal for gossip. But A could also be informal if the supervisors know each other well, and conduct their business in a friendly manner. It’s hard to know if A is right, so it’s best to hold onto it and see if the other answer choices are wrong.
B works as a situation in which gossip could take place, since B is an informal setting. It is also likely that the elderly relative is either “absent or treated as absent.” I can therefore eliminate B.
C also seems like it describes a situation that is gossip-friendly, since talk amongst a group of friends is likely to be informal. Here, the friend’s spouse is the absent person. That’s enough for me to eliminate C.
D is another informal setting, with two people talking about someone who isn’t there. That makes D incorrect. And since A is the most formal setting, I can feel good about choosing it as the correct answer.
The author’s discussion of why gossip can be inappropriate takes place in paragraph 4: “the informality and the speculative nature of what is said may be inappropriate, as it would be if gossip were the basis for firing people from their jobs. The communication about other persons may be of a degrading or invasive nature that renders it inappropriate. And the talk about persons in their absence is sometimes of such a nature as to require that it either be spoken to their faces or not spoken at all.” So there are a few reasons why gossip may be degrading here. It might be too speculative—that is, something not yet confirmed to be true—or it might be too degrading, or invasive. One of those has to be the right answer.
Benefiting from gossip isn’t mentioned as one of the reasons why gossip might be inappropriate, so I can safely eliminate A.
This sounds almost like my first prediction, but B also makes me rethink that part of paragraph 4. There, the problem is not just that the gossip can’t be confirmed; the problem is also that the unconfirmed gossip could have consequences: “if gossip were the basis for firing people from their jobs.” Since B doesn’t resemble that part of the passage, I can eliminate B.
I don’t see anything in the passage which suggests the enjoyment of gossip is somehow wrong, so I can eliminate C.
D is similar to B, but it includes what I discerned that B lacked: some sense that gossip could have negative consequences for its subject. D is therefore correct.
This is an analogy question; I’m looking for an answer that resembles gossip. To answer this question, I need to pull from paragraph 3, where gossip is defined. I’ve done this before in this question set, but analogy questions are tricky, and so I would benefit from being doubly sure of what the passage says: “I shall define gossip as informal personal communication about other people who are absent or treated as absent. It lacks formal rules setting forth who may speak and in what manner and with what limitations from the point of view of accuracy and reliability. It takes place spontaneously and relies on human and guesswork, and it is casual with respect to who ends up receiving the information.” So I’m looking for a choice that describes nonspoken communication about someone else that is informal, without a concern for accuracy or discretion.
A is way too formal to qualify as gossip, so I can eliminate it.
B is potentially informal enough in the sense of form, but a political cartoon is still a formal art form: one published in newspapers. That also means that B does not describe a “spontaneous” form of communication—since publishing takes planning—so B has to be wrong.
Even though C describes someone talking about someone who is not present in the conversation, a lineup is also a very formal setting, and is definitely not spontaneous. So I can eliminate C.
D lacks the shortcomings of the other answer choices. People reacting to a lecturer are likely to react spontaneously, in ways that treat the lecturer as if the lecturer is not present. D even fits the part of the description of gossip that mentions how gossip can be “casual with respect to who ends up receiving the information,” since no matter how covert those gestures and grimaces are, those expressions are all potentially detectable by other audience members (or even the lecturer!). That’s enough to make D correct.
Ah, a nice straightforward main point question. The passage begins with a definition of gossip that takes up the first three paragraphs of the passage. Then, the author goes on to explain what is problematic about gossip. Specifically, while the author notes that “Not all gossip, as I have defined it, is injurious or otherwise to be avoided. But when it is, it can harm all who take part in it.” But rather than stop there, the argument progresses one more step: to explain why we gossip in spite of these problems. Paragraph 6 begins with the claim that “Gossip can be an intoxicating surrogate for genuine efforts to understand.” The author’s problem with gossip then reaches its final definition: “Those whose casual talk stops at no boundaries, leaves no secret untouched, may thereby shut themselves off from the understanding they seem to seek.” Since this is where the author’s argument ends, and where it leads to, this has to be the main idea: that gossip may keep people from understanding those they want to understand.
A matches my prediction: that gossip can prevent understanding. I can choose it and move on.
The author actually notes the benefits of gossip in the first two paragraphs, so I can eliminate B.
The rest of the passage suggests that gossip can be harmful, so C has to be wrong.
And D goes further than the passage does. The author notes that gossip can be a problem, but that’s not the same as suggesting that it tends towards such problematic behaviors: that its natural inclination is towards manipulation and denigration. I can therefore eliminate D.
The list of reprehensible forms of gossip are at the end of paragraph 5: “gossip in breach of confidence, gossip the speaker knows to be false, and unduly invasive gossip.” But that just tells me what kinds of gossip are reprehensible: not why they are reprehensible. So my next step is to turn to the paragraph idea: that while not all gossip is bad, “when it is, it can harm all who take part in it. Out of respect for oneself as much as for others, therefore, it matters to discern such cases.” In other words, these three forms of gossip are not just bad for the subject of gossip: they also hurt the gossipers in some way. That should be enough for me to find the right answer.
A focuses on the subject of gossip, but I’m looking for an answer choice which refers to the gossipers, so A has to be wrong.
B has the same problem as A: it is focused on the subject of gossip, and not on the gossiper.
C is definitely focused on the gossiper, and in a way that suggests that the gossip hurts gossipers: by revealing moral weaknesses. C is therefore likely right.
D is really close to C, but the differences between them make C a better answer. C describes “someone who contributes to such talk”: someone who is actively gossiping. D only talks about someone “who listens to such talk”: someone who is not so clearly participating in the gossip. Since D is not as clear a case of gossipers getting hurt by their gossip as C is, D has to be wrong.
I know that the ultimate effects of gossip are discussed in the last two paragraphs. In paragraph 6, the author states that gossipers “may then become unable to think of other human beings in other than trivial ways. If they cannot attribute scope and depth and complexity to others, moreover, it is unlikely that they will perceive these dimensions in themselves.” So one consequence of gossiping seems to be one’s inability to see oneself as a whole human being. Paragraph 7 then says “Those whose casual talk stops at no boundaries, leaves no secret untouched, may thereby shut themselves off from the understanding they seem to seek. Gossip can be the means whereby they distance themselves from all those about whom they speak with such seeming familiarity, and they may achieve but spurious intimacy with those with whom they speak.” So another consequence is that a gossiper’s relationships with others could become impoverished. That gives me two possible right answers.
Neither of my predictions mentions needing to rely on the opinions of others, so I can eliminate A.
B matches my first prediction from paragraph 6: that “If they cannot attribute scope and depth and complexity to others, moreover, it is unlikely that they will perceive these dimensions in themselves.” Since B is just another way of saying that, it must be correct.
The author definitely says that gossip can trivialize others, but C is more exact than that. Since the author is so vague in the passage, I can’t be sure that the trivialization of others includes trivializing their motives, so I have to eliminate C.
D is also mentioned in paragraph 6, but the author builds on the point about stereotypes in order to make a different point about how gossiping may encourage one to stereotype oneself. Since the author builds on D, it can’t be the “ultimate effect” of gossip, so D has to be wrong.
The passage reference to the first paragraph is an introduction to gossip. After mentioning how bad gossip can be, the author goes on to say “Yet to define it in these ways is to overlook the whole network of human exchanges of information, the need to inquire and to learn from the experience of others and the importance of not taking everything at face value.” Since this is an assumption question, I’m looking for something unstated that must be true for the argument to make sense. To describe gossip as a need, and then to describe it as a way of gaining information about others, I have to assume that there is something satisfying about gaining information about other people. That’s an OK prediction, but I’ll mainly be looking for the right answer by testing to see if it is an assumption. If I negate the answer choice and reapply it to the passage argument, the right answer will make the passage argument nonsensical.
I can eliminate A because the desire for knowledge about others is just a desire for knowledge; it is not clearly a desire to help others. I can confirm this by testing the assumption; if I said that humans are not concerned with helping others to understand or to deal with their problems, the argument is unaffected. I can still clearly understand why people gossip, and what the effects of gossip might be.
I also don’t see anything about trying to exploit others, so I can eliminate B.
C is basically the same choice as B, so it’s also wrong.
D matches my prediction: that if we have this “need to inquire and to learn from the experience of others,” to gain information, then fulfilling that need is, by definition, satisfying.
We tend to think of gossip as a bad thing; the author acknowledges this, mentions some positives to gossip, and then proceeds to talk about a specific negative aspect of gossip (how it encourages people to misunderstand themselves and each other). So I’m looking specifically at the part of the passage that discusses the benefits of gossip: what it adds. This starts in the first paragraph, where the author claims that to just define gossip as a bad thing “is to overlook the whole network of human exchanges of information, the need to inquire and to learn from the experience of others and the importance of not taking everything at face value. The desire for such knowledge leads people to go beneath the surface of what is said and shown and to try to unravel conflicting clues and seemingly false leads.” That suggests that gossip helps me to learn about others and from others. That should be enough to find a right answer.
I don’t see anything about social advantages in that quote, or in paragraph 2. So I can eliminate A.
The author brings up shaming others as a potential downside to gossip in paragraph 4. Since B does not describe a potential benefit to gossip, I can eliminate B.
This sounds compelling. After all, isn’t part of learning about other people becoming intimate with them? But to justify C, I have to add that statement; it’s not something that is stated in the passage itself. That makes me fairly unsure about C, so I’ll hold onto it, but expect that D will be better.
D is definitely better. An implicit part of learning about others and from others is learning about one’s relationships to others. This also aligns with the passage’s main idea, since it is this relational dimension to gossip that can both be beneficial and harmful. I can therefore choose D and eliminate C.
This question is worded strangely; the difference between what and what? Luckily, the reference to the passage helps. The Greek concept of rage is mentioned in the first paragraph: “The first word of The Iliad is menin—in old Greek, ‘rage’ or ‘wrath’—the kind of rage that has an element of divine fury in it and destroys armies and breaks cities.” The paragraph then contrasts this depiction of rage as somehow divine with our own view of Achilles in the next sentence: “But to us, Achilles’ anger seems less divine than egotistical.” In other words, the major difference is that to us, Achilles’s anger is more selfish, and less divine or holy.
A matches my prediction; selfishness, when contrasted against the divine, is synonymous with being human. I would choose A and move on.
I don’t see any mention of human nature around where the Greek concept of rage is mentioned, so I can eliminate B.
There is definitely a suggestion that Achilles is being excessive, but I don’t see anything about restraint here, so C has to be wrong.
I can eliminate D because these two concepts aren’t contrasted in the passage. Instead, they are synonymous with each other; booty is one way that glory is gained according to paragraph 6, where the passage implies that the Greeks held an idea of honor which depended “on the bartering of women and goods.”
This question is asking me to account for how the passage will react to new information. How would the ancient Greeks, as depicted by the author, react to the new information? I think it would seem alien to them because of how the author describes Greek civilization in paragraph 4: “the Greek view was savage, but it was offered without hypocrisy. Accepting death in battle as inevitable, the Greek and Trojan aristocrats of The Iliad experience the world not as pleasant or unpleasant, or as good and evil, but as glorious or shameful—a noble rather than an ethical conception of life.” So I’m hopefully looking for an answer that mentions that the Greeks would disagree with the notion of fighting a war to prevent a bigger war, because a bigger war would be an opportunity for greater glory.
The passage author argues the opposite: that while we see the world—and therefore probably war—as ethical, the Greeks did not. They saw world through terms of nobility, not ethics. So A has to be wrong.
B is immediately attractive because it matches half my prediction by mentioning Greeks considering war in terms of glory. That is the underlying reasoning for why the Greeks would disagree with the idea of war in the question stem. The opposition is also clear insofar as it makes sense for the people who hold the view in the question stem to view war in moral terms—that a larger war is morally bad—while the Greeks may have looked forward to a larger war. I am almost certain that B is correct, but it makes sense to also double check the other answer choices, since the answer choice didn’t take a form I was expecting.
I can perhaps see how the information in the question stem might be viewed as heroic—insofar as it is heroic to stop a larger war from happening by fighting a war—but that is not a context that the Greeks would share, as the Greeks are depicted in the passage. This is part of what the author is so concerned about with Achilles: that it only makes sense for him to be a hero if the Greeks held a very different view of heroism than we do. So C has to be wrong.
The information in the question stem doesn’t mention the worth of human life, so there’s no reason why it would affirm the Greek belief about the worth of human life. D is therefore wrong.
I know that paragraph 6 argues that Achilles’ sense of war changes. As the second sentence of that paragraph states, “Suddenly, he is groping toward an idea of honor that does not depend on the bartering of women and goods or on the opinions men have of one another’s prowess.” These ideas get crystallized by the end of the paragraph: “Achilles has come close to breaking with the honor/shame code of Homer’s warrior society.” Since this claim comes after the quote, I am therefore safe in assuming that this is what the quote is meant to support.
A matches my prediction, so I can choose it and move on.
Achilles’ anger is mentioned in paragraph 5, but the author shifts the argument and says that the anger itself is meaningful for other reasons. So B is incorrect.
This is a distortion of the quote. I can see how someone could read the quote as suggesting that those who do a lot are equal to those who do nothing in the face of death. But the author never says that Achilles believed that people deserve equal rights. Instead, the author suggests that Achilles comes close to the idea that some people are worth more than others, which is a claim about worth, but not rights. C is therefore wrong.
There’s almost no mention of gods or faith in the passage, and there is none in paragraph 6, so I can eliminate D.
The passage is almost totally focused on talking about Achilles and the ancient Greeks, which makes the discussion of Western culture in paragraph 4 so jarring. But I know from my initial reading that the passage brings up Western values—specifically, how “The Western reader comes from a society that is nominally ethical”—in order to contrast with what Greek society valued. That should be what the right answer says.
Paragraph 4 argues the opposite: that Western culture is likely more moral than Greek culture because Western culture is an ethical culture, while the Greek culture was noble. So A has to be wrong.
B is also the opposite of what the passage argues. The author mentions Western culture to bring up how different it is from Greek culture, not how similar it is.
This doesn’t exactly match my prediction, but I can see how C could be correct. By contrasting with Greek society, the discussion of Western culture helps me to better understand Greek culture. But while I think C is correct, it’s different enough from my prediction that I should eliminate D just to be sure.
D is definitely wrong because the passage doesn’t try to talk about how The Iliad is seen in all cultures. C has to be correct.
The idea behind Achilles’ conclusion about the value of life is that it challenges the warrior code of his society: the honor and glory one gets from fighting. So I can predict that the right answer would have to mention something about finding glory in war.
A isn’t about war, so I can eliminate it.
B has the same problem; it too isn’t about war, so B is wrong.
C is about war, but I can eliminate it because protesting against war would attack the notion of finding glory in war. C is something that someone who accepted Achilles’ view of life is more likely to do, not less.
D, though, matches my prediction, since it talks about benefiting from fighting. Since Achilles was moving away from an understanding of the world that honored killing enemies, D has to be correct.
This is a difficult question to start the passage off with, because the whole passage compares pseudo-events and events. It is not until paragraph 5 that a straightforward comparison between the two is made: where the author mentions that the pseudo-event’s “relation to the underlying reality of the situation is ambiguous […] While the news interest in a train wreck is in what happened and in the real consequences, the interest in an interview is always, in a sense, in whether something happened and in what might have been the interviewee’s motives. Did the statement really mean what it said?” The difference between the train wreck and the interview/pseudo-event is that the train wreck story confirms reality, while the interview seems to make reality more questionable, not less. That should be enough to find the right answer.
I don’t get the sense that the passage talks about seriousness enough for A to be correct. While a train wreck is clearly serious, an interview can be too.
The passage seems to imply that pseudo-events are staged for the public and for the sake of publicity. That implies that pseudo-events do have and generate interest. So B has to be wrong.
C matches my prediction, since to say that pseudo-events lack certainty is to say that they seem to make reality more questionable.
There are clear results in the pseudo-event of the hotel part in paragraph 2: to increase the publicity of the hotel. So D has to be wrong.
To answer this question, I need to deduce what the passage does. I know that it begins by arguing that pseudo-events exist, and then goes on to explain what a pseudo-event is. That’s actually what the rest of the passage covers, in various ways. So I’m looking for an answer choice which mentions that the passage is trying to explain what pseudo-events are.
The passage does define pseudo-events, but there are barely any events or references to the past: elements that one would think would be necessary if the passage was trying to provide a history of pseudo-events. The one major historical reference—to Napoleon—isn’t even to a pseudo-event. So A has to be incorrect.
If B were correct, I would expect a list of kinds of pseudo-events. Since the passage does not focus on providing one, I can eliminate B.
I can eliminate C because I actually don’t know much about the effects of pseudo-events based on the passage. One would think that if C were true, there would be more to say about those effects.
D is the last choice, and it is correct. To clarify the nature of pseudo-events is another way of saying that the passage explains them. I can choose D and move on.
In order to ascertain what would have displeased the organizers of the hotel’s anniversary celebration, I need to be sure I know what would have pleased them. Paragraph 2 claims that the pseudo-event was staged to “increase their hotel’s prestige and so improve their business.” So I’m looking for options which would suggest the opposite of either of those outcomes.I. I would be unlikely to either raise the hotel’s prestige or improve its business, since such a story would suggest that the hotel might not be worth visiting. I is therefore correct.

II. I have no idea why II would displease the celebration organizers. The passage doesn’t clearly suggest that a pseudo-event needs to happen within a specific time frame, so II seems out of the passage scope enough for me to eliminate it.

III. I think III is incorrect. If the goal of the celebration is to increase the prestige of the hotel, III sounds like a way to do so in the short term, but that could hurt it in the long term. Since I can’t be totally sure of what effect III would have on the celebration organizers, I should assume III is incorrect.

A is correct. Only I works.
II is wrong because there is no clear indication that it is a problem to report the celebration a week after the event.
III is incorrect, so C is wrong.
Both II and III are wrong, so D is wrong.
Because the question stem asks me about why pseudo-events “continue to be reported,” I know that I probably need to go to the last paragraph, where the author speaks directly about why pseudo-events will keep emerging. There, the author notes that “People expect more of them and are given more of them.” So I’m looking for an answer choice which mentions the public’s desire for pseudo-events.
While the passage does differentiate between pseudo-events and normal ones, I don’t remember the passage ever suggesting that the media was supposed to distinguish between them. If that is not their responsibility, then A can’t be right.
B matches my prediction, insofar as it suggests that the media is reacting to what people want. B is therefore correct.
C is really similar to A, insofar as the passage seems to suggest that a newsworthy story is authentic, so a planted story might not be. The two are similar enough that if one is correct, the other one would likely have to be too. That means that both A and C have to be incorrect.
I don’t remember anything in the passage about the media trying to curry favor with anyone, so D is definitely wrong.
This is essentially a weakener; the right answer will attack the way the author attacks interviews. The author’s attack on interviews takes place in paragraph 5: “While the news interest in a train wreck is in what happened and in the real consequences, the interest in an interview is always, in a sense, in whether something happened and in what might have been the interviewee’s motives.” In other words, the author seems to be claiming that interviews do not attempt to ascertain what happened. But what if an interview did? That might challenge the author’s blanket dismissal of interviews as pseudo-events.
A matches my prediction. It is likely that an interview like A would be conducted in order to discern “what happened” and “the real consequences” of what happened. That’s enough to suggest that the interview might not be a pseud-event, which in turn makes A correct.
Since B describes an event that hasn’t yet taken place, that leaves B open to the author’s critique of interviews: in this case, whether or not the young athlete will win the competition. Since that fits the author’s argument, B has to be incorrect.
C shares a key feature with B: both refer to events that have not happened yet. Both are therefore wrong for the same reason: because the author claims that interviews tend to focus on whether something happened or will happen.
D doesn’t involve an event that has not happened yet, so D is already better than B and C. But D is wrong because it doesn’t clearly involve someone who can tell what happened at the peace conference. The participating leader is not an authority on the peace conference in the way that a police officer would be an authority on a spectacular disaster. So while D is attractive, it is worse than A, and so is incorrect.
This is such a broad question stem that I have no choice but to just evaluate each option. The wrong answers will not be stated or implied by the passage, while the right answer will.
I can’t choose A because it is not clear that historians have clearer goals than do scientists. I know that they have different goals, which are in some respects more or less difficult than the other, but the passage does not really explain what it means for a goal to be “clearer.” Without more from the passage, then, A has to be wrong.
The author says the opposite: that in some ways “the task of the historian is more complex than that of the scientist. In another way, it is simpler” (paragraph 4). Since the author does not simply say that science is easier to do than history, B has to be wrong.
I would choose C because of paragraph 4: “the events of history are never just spectacles for contemplation; they are phenomena that the historian looks not at, but through, to discern the thought within them.” In other words, historians do not just describe human actions; they try to understand why those actions took place. C resembles the passage enough, then to make it correct.
This is a tricky one. The passage seems to suggest D, but it actually says something more complex in paragraph 3: “In the case of nature, this distinction does not arise.” In other words, it is not that scientists are not concerned with the inside of events. Rather, science does not approach its object of study as if there is an inside or outside. D therefore distorts the passage enough to make D incorrect.
I know that the passage mentions Caesar as an example of what historians study: both the inside and outside of the event, the fact and what motivated the fact. The author clarifies this example in the second paragraph: “They are interested in the crossing of the Rubicon only in relation to Republican law.” The implication there is that Caesar crossed the Rubicon in defiance of Republican law: as if Republican law was supposed to be able to stop him from crossing the Rubicon. So if Caesar did not cross the Rubicon, I can safely assume that he did not do so because of Republican law.
A matches my prediction, so I can choose it and move on.
The passage does not discuss Caesar’s bravery in relation to moving across the Rubicon, so B is safe to eliminate.
The river is barely mentioned in the passage, and it is never described enough for the passage to suggest that crossing it was dangerous to try.
The passage does mention a constitutional crisis, but that mention comes in paragraph 1. It is associated not with the crossing of the Rubicon, but with Caesar’s assassination: “the clash of constitutional policy between him and his assassins.” That makes D incorrect.
The answer choices tell me that I’m going to need to evaluate the new information in the question stem in terms of the passage: how it lines up with passage information. The key word in this question is “motivation.” I know that the passage distinguishes history from science. I also know that the passage suggests that history is all about motivation; the historian’s main task is to discern “the thought of its agent.” I therefore know that science should also not deal with motivation. So I’m looking for an answer that either claims that the social psychologists are in line with historians, or are not in line with scientists.
A matches my second prediction. Paragraph 2 states that historians think about both the fact of an event and its motivation, while paragraph 3 states that scientists simply don’t think of their subject matter in such terms. That makes A correct.
The social psychologist implicitly accepts the distinction between the outside and inside of events by trying to discern motivation. Motivation, as paragraph 1 states, is the thought that goes into an action. So if social psychologists study motivations, they accept an implicit split between events and the thought that goes into them. B is therefore wrong.
C also goes against the passage. The scientific approach does not approach phenomena in terms of motivations according to paragraph 3.
D also contradicts the passage because the information in the question stem shows a sense of cause that historians also share: one based on motivation.
This question refers to a pretty confusing moment in the passage: so much so that I only made sense of it by relying on the passage contrast with science given in the final paragraph. If, for scientists, understanding an event is an occasion to try to discern some natural laws or universal explanations for all events of that type, the author is saying that historians stop once they know why someone did something; there is no broader rule or law to discern.I. The author would not agree with I because historians do discover reasons for events; the consistent example in the passage is the historian’s attempt to understand the cause of Caesar’s actions. So I is incorrect.

II. This is true. What’s so confusing is that we often think of a fact to be what the passage calls the “outside” of an event: what paragraph 1 defines as “everything about it that can be described in terms of persons and their movements.” But in paragraph 5, “fact” is being used to define the intention behind an action. That’s clear in the sentence following the quote: “When historians know what happened, they know why it happened.” So II must be correct.

III. This is a little more difficult to tackle. I know at the end of paragraph 5 that the author of the passage is thinking about cause and effect when talking about history: “When historians know what happened, they know why it happened. This distinction does not mean that the concepts cause and effect are inappropriate in reference to history; it means that they are used there in a special sense.” Since that’s the end of the paragraph, the next paragraph has to explain what that means: “When historians ask ‘Why did Brutus stab Caesar?’ they do not mean ‘what historical conditions led to this event?’ but ‘What did Brutus think that made him decide to stab Caesar?’ The reason for this thought relates it to other events, with their own internal motives.” That sounds a lot like a chain of cause and events, since it talks about relating thoughts to other events. But the last sentence clarifies that the author is describing a different conception of causality: “The cause of an event, for historians, is not something apart from it but its inner aspect.” Instead of a notion of a chain—like billiard balls hitting other billiard balls, one event causing another—the author says that history uses an inner-outer metaphor for causality; that instead of a chain, in which event influences event, there is some internal thought process that affects all different kinds of events. So III is correct.

I is the only incorrect option, so A is incorrect.
II is definitely correct, so B is wrong.
I is incorrect, so C is incorrect.
D has to be the right answer, and it matches my prediction.
This covers the exact same ground as the last question: almost as if the test-makers recognize how challenging this question set might be. Theory formulation is discussed in the last paragraph: “When scientists ask ‘Why did that piece of litmus paper turn pink?’ they mean ‘On what kinds of occasion do all pieces of litmus paper turn pink? What are the invariable antecedent conditions?’” In other words, scientists form theories because there are some “invariable antecedent conditions” that could predictably cause the same effects. Historians, on the other hand, focus on specifics: “they do not mean ‘what historical events led to this event?’ but ‘What did Brutus think that made him decide to stab Caesar?’” So I can guess that the reason the passage author thinks historians don’t formulate theories has something to do with specifics: with how singular historical events are.
A matches my prediction. For the passage to say that science forms theories because of “invariable antecedent conditions” is another way of saying that natural phenomena operate with some predictable certainty. And since the passage consistently contrasts history and science, I know that the author has to imply that history does not operate with that certainty. Also, the idea that historians focus on specific events and do not generalize from them is in line with the notion that human actions are too predictable to justifiably theorize on. Still, this is a tough question, so I would probably eliminate the other answer choices first before choosing A.
The passage does not say anything about historians searching for fundamental principles of history, so I can eliminate B.
Nothing in the passage says that historians don’t do research. Research isn’t really mentioned at all in the passage. So C has to be wrong.
and the present much, so it never suggests anything like D. A has to be the correct answer.
The first paragraph establishes the problem mentioned in the question stem, so the second paragraph explains why the problem is what it is: “The leading source of the exaggerated image of presidential policymaking power is a tendency to overemphasize the personal contribution of past presidents to sweeping policy changes.” So the right answer will mention that presidents get associated with legislation changes.
A matches my prediction, since it is almost a word-for-word quote of the second paragraph. So A is correct.
The passage argues that B is an illusion: that even congresses controlled by the president’s party often vote against the president. So B has to be wrong.
This is never mentioned in the passage; I know that the opposite is, though, in paragraph 6, so I can assume that C is a wrong answer choice that distorts the passage argument.
The passage is very clear that D is false: The U.S. legislative system is not a parliamentary system. D is therefore wrong.
This echoes a claim from the second to last paragraph, where the author argues that the role of the federal government has been changing: “Suspicion in the aftermath of repeated ethical scandals involving presidents has led to increased congressional surveillance of the White House.” This is part of a larger argument about how hard it can be for presidents to get legislation passed. That should be enough for me to find a correct answer.
The distorted image of the presidency has nothing to do with the Watergate scandal as described in the question stem. The public image of the presidency is explained in paragraph 1: that “it ought to function with the kind of authority over the policy process that a prime ministry has in a parliamentary system.” But the Watergate example in the question stem doesn’t have anything to do with policy processes. So A has to be wrong.
B is wrong for the same reason that A is wrong: because B is made in the context of legislative control, or getting legislation passed. That has nothing to do with unethical behavior by the president, so B has to be wrong.
The passage never suggests that prime ministers could get away with ethical misdeeds, so C has to be wrong.
D works as the correct answer, since it is a reasonable entailment of paragraph 6. The Nixon incident is an ethical scandal that “had led to increased congressional surveillance of the White House”: surveillance that is only necessary because of “congressional mistrust of the president.” I should choose D and move on.
The author’s description of how a parliamentary government functions occurs in paragraph 3: “In a parliamentary system, responsible government means a fusion of the executive and legislative powers rather than a separation. It requires the government leaders—a prime minister and other cabinet ministers—to present a unified front to the legislature. And if the legislature refuses to support the government on important policy matters, a prime minister can either immediately dissolve the parliament or resign along with the rest of the cabinet.” The right answer will need to accord with this description.
A is incorrect because paragraph 3 says that if the legislature opposes the prime minister, that will necessitate “a new election, which could jeopardize the political careers of some members of the parliament.” Since the passage only mentions that new elections could hurt politicians, there is not enough in the passage to make A correct.
B is a reasonable explanation. It makes sense generally, since the author uses the parliamentary system to contrast with the American one. If American presidents have a hard time getting legislation passed because the government does not need to vote as the president dictates (paragraph 4), parliament must logically encourage the government—which includes the ministers—to vote with the prime minister. B is also stated as a requirement of parliamentary government in paragraph 2: in a parliamentary government, “It requires the government leaders—a prime minister and other cabinet ministers—to present a unified front to the legislature.” Since that is stated in the passage, B must be true.
I only know from paragraph 2 that one parliamentary leader—the prime minister—could offer to resign if one of their proposals are outvoted. I have no idea if multiple leaders would offer to resign. So C has to be wrong.
I don’t have any passage information regarding how parliament would react to a scandal involving the prime minister. It’s just not discussed in paragraph 2, so D has to be wrong.
This question requires me to know the difference between how presidents and prime ministers behave, and to be able to apply my knowledge of this difference to an analogous situation: teaching. I know from my previous work that a prime minister gets to exert a lot more control over the government, while the president does not, so the right answer will have to say something about the professor not directly controlling the class. This is most clearly explained in the first paragraph: “Congress now awaits the president’s proposals, for the president has the task of establishing a legislative agenda and setting its priorities. But the image of the U.S. president as a powerful chief legislator is an illusion.”
This is more like a prime minister than a president, because the passage focuses on how weak a president actually is. Since A instead focuses on how “authoritative” the professor is, A can’t be analogous to the passage descriptions of the professor. That makes A incorrect.
I know that the president sets a legislative agenda (paragraph 1), but the passage repeatedly argues that presidents often have trouble getting the legislature to follow that agenda (paragraph 4). So B has to be wrong.
While the passage does talk about how weak presidents are, the passage does not suggest that presidents delegate all their authority to others. C goes way too far, so it has to be incorrect.
D works as the right answer. A professor who tries to guide students follows the same logic as a president who tries to guide the legislature. An open discussion is a forum in which professors have cede a lot of control over to students, just as a president lacks control of the legislature.
Changes to the U.S. government are mentioned in paragraph 5: “In recent years, the gap between the promise of legislative leadership and what can realistically be achieved grew even wider. Presidential misjudgments, as in the Vietnam intervention, prompted action by Congress against the president’s foreign-policy authority, and the domestic-policy environment has since grown increasingly hostile to direction. A decline of political party influence has contributed to a dispersion of power within Congress, making it more difficult for presidents to work through party leaders to develop and steer a legislative agenda through the Senate and House.” The right answer has to mention one of these reasons.
A is incorrect because the passage primarily focuses on the relationship between Congress and the White House. A doesn’t immediately involve Congress, so A has to be wrong.
Paragraph 5, where changes are discussed, focuses on the opposition between Congress and the president. So world leaders would not play a role in the changes mentioned in the question stem, which makes B incorrect.
gain, the right answer has to address problems between Congress and the president. C only talks about the media and proposed legislation; it does not clearly involve the legislature, which makes C incorrect.
D matches my prediction because it is synonymous with the end of paragraph 5. One way that “political party influence” goes into decline is when votes in Congress grow more independent of party agendas. D has to be correct.
The best way to answer this question is to look for something that the author disagrees with. There are two related disagreements that start off the passage. The first is in the first paragraph, where the author claims that while other scholars view prehistoric naturalism as a “merely instinctive static phenomenon,” the author claims that this form of art “already reveals all the typical phases of development through which art has passed in modern times.” The second disagreement is something of an illustration of the first. This one is given in paragraph 2, where for the author to claim that “There are no parallels whatever between this prehistoric art and child art,” there must be people who equate the two. So I’m either looking for an answer choice which mentions that prehistoric art is not childlike, or one that mentions how prehistoric art went through typical steps in the development of art.
A is a great answer choice, because it is an almost exact match for my prediction from paragraph 1. I would choose A and move on.
There is no clear disagreement with this idea. The author definitely agrees with it in paragraph 4—“We know that it was the art of primitive hunters”—so B has to be wrong.
This is also brought up in paragraph 4, but as a question: as something unsettled. That means that the author is not sure enough of C to argue against anyone about it, so C has to be wrong.
This is an attractive answer choice because it is mentioned in the first paragraph, alongside the information that confirms the right answer, but note that the author suggests that other experts agree with this claim. To say that “The most remarkable thing about prehistoric naturalism is not that it is older than the geometric style” is to suggest that the author’s point of disagreement with other scholars will not involve this fact, but something built on it. That makes D wrong.
Children’s art is mentioned in paragraph 2, where the author says that such art is “rationalist, not sensory: They show what the child and the adult artist know, not what they actually see; they give a theoretically synthetic, not optically organic, picture of the object.” So that tells me two things: that children’s art suggests something about how children think, but it does not serve as a good indication of what children actually see.
This is a choice that is meant to tempt me to bring in outside information to justify this answer choice. We typically think of children’s art as direct and unsophisticated, but the author does not state this, and so A has to be wrong.
B is wrong because the author says the opposite: that children’s art reveals how a child thinks about the subject of art, not what the child actually sees.
C is designed to be an unappealing answer choice, because we don’t typically think of children as employing and possessing reasoning and knowledge, respectively. But C has to be right, because it is more than close enough to the claim that children’s art is “rationalist, not sensory.” In other words, since C says that children’s art reveals how children think, C has to be right.
D is definitely wrong. The author doesn’t quite use these terms, since the author is invested in explaining exactly how children’s art is subjective—whether it presents the child’s observations or the child’s thinking—and not whether or not the child’s drawing is truthful. Even if I accept that the child’s faithful representation of an observation is truthful and objective, D is still wrong, since the second paragraph claims that children do not depict what they see in art.
This is a challenging question because the passage does not directly mention modern artists. Modern times are mentioned in paragraph 1, but I shouldn’t equate people with a period, so that’s probably not where I need to go to answer this question. Modern people, however, are mentioned in paragraph 3, so that might be helpful: “The painters of the Paleolithic Age were still able to see delicate shades with the naked eye that modern people are able to discover only with the help of complicated scientific instruments.” Since I can’t see any other references to anything that even remotely resembles a reference to modern painters, I’ll try this out as my prediction: that painters of the Paleolithic Age were better at seeing the world than modern painters are.
Paragraph 3 tells me the opposite: “The peculiar thing about the naturalistic drawings of the Old Stone Age is, on the other hand, that they give the visual impression in such a direct, unmixed form, free from all intellectual trimmings or restrictions.” That suggests a depiction with great visual fidelity, and so A has to be wrong.
B refers to a different aspect of the same concept that A does: how the painters of the Old Stone Age depicted what they saw. B is therefore also as wrong as A is, since paragraph 3 argues that those painters were better able to see their environment and capture it in their paintings than we are able to do naturally.
To say that these Paleolithic painters painted “in such a direct, unmixed form” is to say that they did not paint abstractly. So C is definitely wrong.
D is the last answer, so it has to be correct. It also matches my prediction; to say that these Paleolithic painters were more aware of their visual environment is to say that they saw the world better than we do (and so, presumably, better than modern painters do).
This question is asking me to locate an apparent or actual contradiction in the passage by giving me one term of that contradiction: the discussion in the final paragraph of what prehistoric art was for. There, the author says that “The pictures were part of the technical apparatus of this magic.” So I need to find a claim in the passage which argues against the idea that these paintings were magical. This brings me to the fourth paragraph: “We know that it was the art of primitive hunters living on an unproductive, parasitic economic level, who had to gather or capture their food rather than produce it themselves, who to all appearances still lived at the stage of primitive individualism in unstable, almost entirely unorganized social patterns, in small isolated hordes, and who believed in no gods, in now world beyond life and death.” To clarify this further, the fifth paragraph states that “In this age of purely practical life, everything obviously still turned around the bare earning of a livelihood, and there is nothing to justify us in assuming that art served any other purpose than a means to the procuring of food.” There is at least a paradox here: how, on the one hand, these paintings hint at nothing beyond the mundane and everyday pursuit of food, and how, on the other hand, these paintings are magic, which is a word we don’t use to describe the mundane. Note that this is a paradox, and not a contradiction—the author would likely respond to my concern by saying that this use of magic is purely about food—but a paradox is all I’m looking for, since I’m searching for something that “appears contrary to the assertion” about prehistoric art, and not something that definitely contradicts that claim.
A is mentioned as part of the explanation of the purpose of prehistoric art. In the last paragraph, the author argues that part of the way the magic worked is by being “both representation and the things represented, both wish and wish fulfillment.” In other words, hunters painted successful hunts because doing so would let them find successful hunting instances in reality. Since A is definitely not contrary to the author’s discussion of the purpose of prehistoric art, A has to be wrong.
B is a possible candidate for the right answer choice. To say that prehistoric painting was a “technique without mystery” sounds like an acknowledgement of the mundane roots of such paintings in the search for food in a world with “no gods” or a “world and life beyond death.” It also contradicts the last paragraph, in which there is mystery in such paintings insofar as magic is involved. But this feels like a slight stretch, and so I’m reluctant to choose B right away. I’ll instead hold onto it and double check the other two answer choices to confirm they are incorrect.
C is actually perfectly in line with the last paragraph, since part of the reason why these ancient painters painted was to use magic in order to capture food. That’s enough in common with the passage to make C wrong.
D is actually very similar to C, since to say that painting was a weapon in the struggle for livelihood is also to say that these paintings were used to capture food. That similarity is enough to make both C and D wrong, which in turn makes B correct.
The only part of the passage that seems to speak to the beliefs of Paleolithic artists is the last paragraph. I know from the last question that the author claims that these painters believed in a sort of magic, that by painting successful hunts, they would find success on hunts: “The paleolithic hunter and painter felt in possession of the thing itself in the picture, felt power over the object in the portrayal of the object. The hunter believed the real animal actually suffered the killing of the animal portrayed in the picture.” That suggests a belief amongst Paleolithic artists in some connection between what they painted and what happened in real life.
Nothing like A is mentioned in the last paragraph. A is also probably wrong because it goes against the author’s description of these paintings in paragraph 3: how “they give the visual impression in such a direct, unmixed form” that it is more likely to be seen as objective than subjective.
The description of the paintings seems to suggest that the subject of the painting took on the qualities of the portrayed object, not the artist. The hunter didn’t become what the painting portrayed; the hunted animal did. So B has to be wrong.
C is definitely wrong because nothing in the passage suggests that these painters sought to impress anyone.
D is the last choice, and it is clearly correct. It perfectly matches my prediction, and the passage discussion of the goals of Paleolithic art in the last paragraph.
This question type is asking me to consider what impact, if any, the assumption in the question stem would have on the passage. My immediate reaction to the assumption in the question stem is that I don’t think the passage addresses this. The passage is all about how paleolithic cave paintings were for the capturing of food because they depict the hunting death of animals. But if there are paintings that don’t obviously involve animals, how can these paintings be about hunting food? The right answer is likely to suggest that the assumption in the question stem weakens the passage by suggesting that the cave paintings may have had a different purpose than the one the author proposed.
The passage doesn’t say that the paleolithic artists painted what was important to them. It argues instead that they painted one thing that was very important to them: the hunting of food. That makes A incorrect.
B seems like a good candidate for the right answer since it mentions that the assumption in the question stem would weaken the passage, but I don’t know enough about how paleolithic hunters saw mountains, lakes and rivers to justify choosing B. The passage never says that the subjects of cave paintings had spiritual powers, so why would the subjects of the paintings described in the question stem have such power? B therefore has to be wrong.
C matches my prediction. If the author claims that the cave paintings were there so hunters could find food, there is no obvious way that the cave paintings in the question stem could have been painted with a similar end in mind. That’s enough to make C correct.
D has to be wrong because the information in the question stem definitely affects the passage argument. It is true that the only paintings discussed in the passage are those that depict animals, but the author is talking about all paleolithic cave paintings, and not just those that depict animals.
I know this is in paragraph 5, since this question takes up the entirety of paragraph 4 to ask: “In an age of purely practical life, everything obviously still turned around the bare earning of a livelihood, and there is nothing to justify us in assuming that art served any other purpose than a means to the procuring of food.” The reasoning for this is provided in paragraph 4: “We know that it was the art of primitive hunters living on an unproductive, parasitic economic level, who had to gather or capture their food rather than produce it themselves, who to all appearances still lived at the stage of primitive individualism in unstable, almost entirely unorganized social patterns, in small isolated hordes, and who believed in no gods, in no world and life beyond death.” So the right answer has to mention how these painters lived in a world that required them to think about food constantly: a world so threatening and unstable that food production had to be first and foremost in their minds.
If anything, paragraph 4 seems to suggest the opposite of A: that these hunters would have lacked leisure time, since they needed to hunt in order to survive. That makes A wrong.
B matches my prediction, and the claim from paragraph 4. It’s clear that people who need to survive would have to “gather or capture their food rather than produce it themselves,” and would be living in a state of instability. B has to be right.
There is no comparison made in the passage between skill in hunting and skill in painting. All I know is that the hunters believed that what they painted would come true; that doesn’t suggest anything about the quality of the paintings or the quality of the hunt. C is therefore wrong.
D is immediately wrong because it begins with a phrase that plays almost no role in the passage: “social organization.”
This sounds like a main idea question, but because the question tells me I’m looking for a claim about advertising which “corresponds most closely” to the author’s thesis means that this is closer to an analogy question. It’s asking me to explain the passage argument in terms of advertising. I know that the passage is about Americans and how they individuate themselves through purchases. This is established in paragraph 2: “American has no native signals to instantly communicate status. So precisely because the U.S. is a democratic nation, the demand for elitist goods is particularly strong among its citizens.” The role of advertisers in U.S. society is outlined in the third paragraph: “U.S. advertisements are not sources of product information but exercises in behavior modification. Appealing to subconscious emotions rather than to the intellect, advertising campaigns are designed to exploit the discontent induced by the dream.” I’m not sure I have a fully formed prediction of the right answer here, but that should be enough information for me to recognize the right answer when it comes up.
I don’t see anything in the passage discussion about unfair competition, so I can eliminate A.
I also don’t see anything specifically linking advertising to prosperity, so I can eliminate B.
Communism is very briefly referenced in the passage, but I don’t really have enough information about socialist nations to be sure that advertisements operate differently in such nations. Also, since socialist nations are barely mentioned in the passage, it’s highly unlikely that the author’s thesis would have anything to do with C, so C has to be wrong.
D is the last answer, but it relates well to the parts of the passage I referred to. Specifically, D resonates with the claim from paragraph 3 that “advertising campaigns are designed to exploit the discontent induced by the dream.” D is therefore correct.
Woody Guthrie is mentioned in paragraph 2: “’This land is your land, and this land is my land,’ Woody Guthrie’s populist anthem tells Americans, but Americans keep trying to increase the ‘my’ at the expense of the ‘your.’” To call the song an “anthem” is to suggest that the song is important to Americans, but the sentence describes how Americans behave in a way that runs counter to the ethos of the song. So the right answer will have to say something about how the ideals of the song are not reflected in the American society that values the song.
A works. To say that the song is an inaccurate anthem is to both suggest that the song is important to Americans, but that Americans do not follow its wisdom. I would choose A and move on.
There is no sense in the passage that the author considers the song to be a parody. The passage actually requires Guthrie’s song to be serious, since the passage is arguing that Guthrie’s song says something about the disconnect between how Americans see themselves and how they really are. B is therefore wrong.
C is definitely wrong. Advertising campaigns aren’t mentioned until later in the passage. More importantly, Guthrie’s song is portrayed in a way that opposes the consumerist culture that advertising campaigns are a part of.
If anything, Guthrie’s song about how we all own the same land seems in sync with de Tocqueville, who said that Americans “are nearly alike, and all follow the same track.” Since both Guthrie and de Tocqueville focus on how similar Americans are, D has to be wrong.
A myth is mentioned in paragraph 3: “the paradoxical nature of the U.S. myth of equality is especially clear in the images that U.S. advertisers use to manipulate the public.” I know that advertising appeals to “the demand for elitist goods” to help “communicate status” (paragraph 2), and that advertising is part of a “consumer economy” that “runs on desire.” The right answer has to refer to at least one of these.
This sounds right, since “professional manipulators” sounds a lot like advertisers, as the passage describes them. But the fact that advertisers manipulate Americans is not enough to suggest that American society is controlled by advertisers. That’s a very strong claim, and one that the passage doesn’t clearly argue, so A has to be wrong.
B is a strange answer choice; one that is hard to choose and eliminate on first glance. I can see how the myth of equality might be responsible for a desire for superiority; paragraph 2 states that “because the U.S. is a democratic nation, the demand for elitist goods is particularly strong among its citizens.” But how does the myth of equality encourage a desire for acceptance? That might be a reference to the first paragraph—to say that “modern America celebrates mass culture, conformity and popularity” is to suggest that there is some cultural value in acceptance—I’m not especially confident in B. So I will hold onto it and check if C and D are wrong.
This sounds like the author’s description of guilt ads in paragraph 4, but the author is very specific about what makes people vulnerable to these kinds of ads: “the fear of rejection,” which is not clearly related to the myth of equality. I can therefore eliminate C.
The author definitely suggests that D played a role in the entrenchment of America’s advertising culture, but it is not clearly related to the myth of equality. I can also eliminate D because equality gets really difficult to define when a comparison between a democracy and a communist nation is made.
I know that anti-Communism is mentioned in paragraph 5. Since the mention comes at the end of the sentence, the significance of the anti-Communist moment is explained in the next sentence: “When any intellectual distinction or social eccentricity could result in public denunciation, citizens were keen to conform. Nobody wanted to be ‘guilty’ of smelling sweaty or of having a soiled collar.” So the anti-Communism moment has something to do with wanting to conform, and a fear of being seen as different.
The passage doesn’t actually talk about morality—right and wrong are not the same as guilt and innocence—so I can eliminate A.
B is wrong for the same reason; morality is not a major topic of discussion in the passage, so B is wrong.
C has to be wrong because it doesn’t talk at all about the fear over being different that is mentioned in paragraph 5.
D, however, definitely matches with that paragraph. To say that conformity is a sign of innocence is to say that people lived in a society where “intellectual distinction or social eccentricity could result in public denunciation.” Note that D differs from A and B insofar as A and B refer to actual morality, while D only refers to the appearance of morality: a “sign” of innocence. If being different indicated to others that you were guilty, then the opposite—that conformity indicates innocence—must be true. D is correct.
Though this is a very broad question, the American dream is specifically brought up in paragraph 2: “Instead of fostering contentment with the unprecedented prosperity achievable by ordinary citizens, “the American dream” breeds desire.” So it might make sense to look for an answer choice which either mentions that the American dream does not foster contentment, or that the American dream encourages desire.
A matches my first prediction based on paragraph 2, so I can choose it and move on.
De Tocqueville is only discussed in the first sentence of the passage, and he makes no reference to the American dream. So there’s no good reason to choose B.
I can eliminate C because it distorts the passage argument. Consumer culture definitely utilizes the American dream to keep developing and perpetuating itself, but the passage never says that the American dream was created in order to sell goods. It is rather the tendencies and beliefs inherent in the American dream that seem to suit consumer culture.
D is tempting, because the author is definitely unhappy with the American dream as it is discussed in the passage. But for D to be correct, the passage would need a much more direct statement from the author to this effect. Since the passage does not offer one, D can’t be right.
I remember this moment in the passage because it felt strange to refer to semioticians, especially because test-takers might not know what the term means: “For the semiotician, the paradoxical nature of the U.S. myth of equality is especially clear in the images that U.S. advertisers use to manipulate the public.” Here, semioticians are being posed as experts of some sort: as people who study the images used in advertising and what they mean. My general knowledge also tells me that a semiotician is someone who studies signs: that is to say, meaningful images. So I’m probably looking for an answer which states that the author refers to semioticians in order to draw attention to the symbols or images used in advertising.
A works. By bringing in semioticians, the author implies that people are paying attention to the symbols used in advertisement. I can choose A and move on.
B is meant to be attractive because it uses the word “signs,” but the lack of clear indicators of status is mentioned in paragraph 2. Paragraph 3 begins with the semioticians, who have a very clear object of interest: the images used in advertisements. B therefore has to be wrong.
The passage doesn’t tell me that semioticians reveal logical paradoxes, and I’m not even sure that the author thinks that there’s a logical paradox inherent in how advertisements work in consumer culture. So I can eliminate C.
The passage references semioticians to make a point. It is not clearly aimed at them. If it were, I would expect more appeals to semioticians throughout the passage. So D has to be wrong.
This is a simple passage check question. The author states this in paragraph 3: “The sound of a brass band, ‘some of the sweetest developments of the divine art,’ first captured his interest in more formal music.” So the right answer has to mention brass bands.
No brass bands here.
None here either.
There it is. C has to be right.
And no brass bands here.
The conversation with P. T. Barnum opposes America and Europe. When Whitman asks if Barnum saw anything in Europe that “made him love Yankeedom less,” Barnum says no: “Why, Sir, you can’t imagine the difference. There, everything is frozen—kings and things—formal, but absolutely frozen!” What he means by frozen is exposed in the contrast with America: “Here it is life. Here it is freedom.” In what was is life and freedom the opposite of being frozen? In terms of energy and activity. That has to be the right answer.
While Barnum plays up the ornateness of Europe, the only sense that the passage gives of what Europe lacks is when Barnum calls it “frozen.” That’s not a word that can easily be related to simplicity, so I can eliminate A.
B matches my prediction: that Europe was “frozen” in the sense of lacking vitality. B is therefore correct.
C is meant to be attractive because Barnum states that in America there is “freedom,” which implies that there is not freedom in Europe. The passage also mentions that Barnum visited kings in Europe. However, it is difficult to imagine how “frozen” could be anti-democratic unless one assumed that was the case already. More importantly, the passage describes art and poetry, and so it would not make much sense for the passage to refer to forms of government when it could be referring to more humanistic forces, like B.
There is no obvious way that “humility” would be the opposite of “frozen,” so I can safely eliminate D.
I know Whitman’s musical tastes are discussed in paragraph 3. There, the author claims Whitman like brass bands, “massed sounds—groups of instruments rather than single ones,” and “popular family vocal groups.” The right answer will likely refer to one of those. It might also be helpful to keep the list of kinds of music from paragraph 4 in mind as well, since that is a list of the music that Whitman disliked: “solo instruments,” “the affectations of the celebrated virtuosos, music that was “anti-republican,” and “the florid Italian and French performance style he heard at Palmo’s opera house.”
A doesn’t sound like any of the options on my list. At best, A does not resemble any form of music mentioned in the passage, and at worst it resembles some of the music Whitman claimed to dislike in paragraph 4, since such operas were likely not American enough for Whitman. That all makes A incorrect.
B runs into a similar problem to A. Even though B is likely a “massed sound,” it would be too European for Whitman’s taste.
C is a perfect match for both what Whitman liked and why. Specifically, it resembles his admiration for “the popular family vocal groups. Their vocal music was fit for ‘natural folk.’ It was ‘heart singing,’ the lusty music of the people, the choral music of democracy.” C is therefore correct.
D has to be wrong because it definitely resembles a few items from the list of Whitman’s musical dislikes in paragraph 4. Specifically, it seems to match his description of operas there: “The trills, the agonized squalls, the lackadaisical drawlings, the sharp ear-piercing shrieks, the gurgling death-rattles” are all descriptions of both vocal challenges and atonality.
This is a paradox question: one that is asking me to help explain a seeming contradiction in the passage. Whitman’s dislike of solo instruments is supplemented with an example of a solo instrument he disliked: “the shallow and tinkling piano.” But he speaks favorably of the operas singer Caterina Barilli-Patti: enough to call her voice exquisite. The simplest solution to the paradox is that he just liked flutes: that flutes were not among the solo instruments that he disliked. Because if I don’t believe that, then the comment by Whitman doesn’t make sense.
There’s no clear way that A helps to resolve the paradox mentioned in the question stem, since it doesn’t help me understand why he would like one solo instrument over another when he is described as disliking solo instruments in the question stem. The right answer needs to specifically address a comparison between solo instruments in order to be correct.
B is the simplest answer, and one that matches my prediction. So B must be correct.
I know that Whitman liked brass bands (paragraph 3), but the flute in that case would be a part of the band, and not, therefore, a solo instrument, as it is depicted in paragraph 5. That means C makes the same mistake that A makes: it does not focus on differences between solo instruments enough to resolve the paradox.
D is clearly wrong because it goes against the author’s main idea: that opera played a positive role in Whitman’s work.
I can answer this by drawing on the paragraph that mentions the kinds of music Whitman disliked (paragraph 4). There, the quote from Whitman about art singing lists many musical complaints, but since the question is asking me for what his dislike of art singing was an expression of, I’m looking for a complaint that cannot be reduced to a musical complaint: one that has significance outside of music. The parts of the quote which have that character are his mention of “foreign method,” “its anti-republican spirit, and its sycophantic tainting of the young taste of the nation!” What those all have in common is that they are very pro-American. The author explains this in the sentence prior: “Walt Whitman (1819-1892) came of age in the crude expansionism of Jacksonian America, bursting its geographical bonds, marching to the Pacific.” So I’m looking for an answer choice which mentions this pro-America bias.
A matches my prediction, since one way to say that Whitman was pro-America is to say that he was nationalistic. I can choose A and move on.
B is a musical complaint, so it would be weird to say that his disdain for a kind of music was an expression of his musical preferences. B is therefore wrong because it just doesn’t make much sense as an answer choice.
The passage doesn’t tell me anything about what languages Whitman understood. And Whitman’s complaints aren’t limited to song lyrics; his objections mainly focus on the musicality of art singing, and what the musicality implies. That makes C wrong.
I don’t know if art singing was accompanied by music. The passage definitely doesn’t mention accompaniment when it brings up art singing, so I can eliminate D.
The author brings up Turner’s conception of the West in order to argue against it. This is clear in paragraph 3: “According to the Turner thesis, Western history stood alone. An exciting counter-trend in modern scholarship favors comparative history […] De-emphasize the frontier and its supposed end, conceive of the American West as a place and not a process, and its history has a new look.” Since this claim contrasts with the description of Turner’s frontier in paragraph 1—“The frontier of Frederick J. Turner was a process, not a place”—the right answer has to mention that the author seems to twist Turner’s argument: taking the geographical boundaries it implies and many of its ideas, but changing it into something more productive.
The author is not clearly in favor of Turner’s version of Western history, so I can eliminate A.
The author is, however, very much engaged with Turner’s version of history: to the point of trying to modify it. So B is also wrong.
C, however, works. The author uses so many of Turner’s terms, but just shifts them in order to make a slightly different claim. That’s definitely revisionist enough to make C correct.
This is probably the next most attractive answer choice after C, since the passage will tempt people to think that the author is critiquing Turner, and so must dislike him. But the author’s argument with Turner is gentle, and does not necessarily suggest that Turner is wrong: just that there is a more productive way to view the West. So D goes too far to be correct.
Discussions over bilingualism in multiple communities are a stand-in for conflicts between communities: specifically over what culture should be considered “primary,” since the primary culture gets to determine what languages are taught in schools. I know such conflicts are discussed in the last paragraph: “a struggle for property and profit is typically accompanied by a contest for cultural dominance.” That the disputes in the question stem qualify is clear in the next sentence: “Conquest often involves conflicts over language, culture, and religion.” So, based on the passage, if I am seeing a conflict here over language, it is reasonable to believe it has something to do with “property and profit.”
A sounds right; it even relates to the paragraph prior, which discusses the kinds of race relations that would emerge over worries about bilingual education. However, it always pays to be really careful when it comes to causal arguments on the exam. Whenever an answer choice says something like “X is the result of Y,” that requires an explicit argument or a very strong implication to be correct. I’m not quite sure that “accompanied by” qualifies as a causal claim; it actually sounds very much like it’s not. So I’m going to keep A, but see if a better answer choice comes up.
B is a better answer than A. It doesn’t make a causal claim—instead calling the discussion of bilingualism “an aspect” of a conflict, which sounds very much like “a struggle for property and profit is typically accompanied by a contest”—and “economic dominance” could very easily be a reference to fights over profit and property. So I would now eliminate A and hold onto B, then go on to eliminate the other answer choices just for peace of mind.
I can eliminate C because it definitely has nothing to do with the conflicts between communities discussed in the last paragraph.
D has to be wrong because calling something a “stage of development” refers to a process, while the author is interested in talking about the West as a place. D, in other words, fits with Turner’s conception of the West, not the author’s.
The specific quote from paragraph 3 that this question is referring to is: “The reorganized history of the West concerns a land undergoing conquest and never fully escaping its consequences.” It may be hard to predict exactly what this statement contradicts or argues against—since it could be many things—but if I keep that sentence in mind, the right answer should jump out at me.
The author would not dispute A. Part of not escaping the consequences of conquest, as the next two paragraphs point out, is that the conquering cultures did not fully erase those that were conquered. Both sets of ethnic groups remain, and contribute to the culture of the West (and therefore the Southwest). Since the author would not argue with A, A has to be wrong.
I don’t know what the author’s arguments about past conquest have to do with how increasing population could be a sign of prosperity. The answer to this question has to have something to do with conquest and consequences, and B doesn’t clearly refer to the latter. So I can eliminate B.
C feels like a good answer. To say that the West is a land “undergoing conquest and never fully escaping its consequences” is a statement in present tense: as if the conquest and its consequences are still ongoing. And to say that a conquest is still ongoing is to say that competition is still ongoing. That’s enough to make C correct.
Legitimacy pops up quite a few times in this passage; for example, paragraph 6 mentions that “in the West, efforts to establish the legitimacy of ownership have overlapped efforts to establish the legitimacy of a way of life and point of view.” The author is being descriptive here, which means I can’t be sure of the author’s perspective on legitimacy of property rights. Since I can’t know how the author feels about D, it can’t be correct.
I know that paragraph 5 talks about the effects of conquest on a region: “Happily or not, minorities and majorities occupied a common ground. Conquest basically involved the definition and allocation of ownership (personal, tribal, corporate, state, federal, and international) and the transformation of land from scenery into property.” I also know that the author’s large point about the West is that thinking about the West as a place rather than a process helps show that conquests do not end when a people is conquered: that instead, the effects of that conquest persist among the populations involved so long as those populations still themselves persist. So I’m looking for an answer choice that mentions how long the effects of conquest are, and how they involve multiple populations.
The author almost seems to suggest the opposite: that conquered people still persist enough to be a part of the “contest for cultural dominance” (last paragraph). So I can eliminate A.
While resources play a large role in the passage argument, they are mentioned primarily in relation to who controls resources, as opposed to the environmental effects of that control. B is here to test whether or not I’m bringing outside knowledge into the exam, so I can eliminate B.
C is also here to test if I’m bringing in outside information into the exam. C sounds like a pretty normal description of what happens after a conquest, and while the passage does suggest that some property owners end up with more influence than others, the passage is designed to argue against C. If C were true, then the struggles between different races and communities in the West would not be as hotly contested as they are in the second half of the passage. So I can eliminate C.
D is the last answer, but it matches my prediction well. I know I’m looking for an answer choice which suggests that there are longstanding consequences to conquest, and one that involves multiple communities. Since D fits both of my criteria, I can choose it and move on.
When paragraph 3 mentions “The reorganized history of the West,” it is not at all clear what that is supposed to mean. But that lack of clarity is actually quite helpful, because it tells me that the author will have to explain this term in the next few sentences. The next sentence says “In this respect, it shares features with the histories of other parts of the nation and the world. According to the Turner thesis, Western history stood alone. An exciting counter-trend in modern scholarship favors comparative history; the history of Western America becomes a chapter in the global story of European expansion.” “Reorganized,” then, has to mean something like relating the West to historical events in other parts of the world. This is especially clear when the author mentions that Turner’s thesis involves seeing Western history as isolated, since the author is also challenging and refining Turner’s thesis in this passage. So the right answer has to say something about the influence of other countries on the West.
This point is made later; it builds on the description of the states as reorganized. So I can eliminate A.
B matches my prediction. If Europe impacted the West in this reorganized history, I can predict that a reorganized history involves world events impacting specific regions. B is therefore correct.
By focusing on the West as a place, the passage ends up focusing more on the people who lived in the West, as opposed to the West’s geographic features. I don’t see much in the passage talking about the physical landscape of the West per se, so I can safely eliminate C.
And there is basically no discussion of democracy in the passage, so D definitely has to be wrong.
I know that the author’s main idea is that the West has been a staging ground for cultural conflict over property and wealth. So I would imagine that the right answer would suggest something of the same logic: not just that conflict would continue, but that the conflict will be tied into the dynamics between different groups in the area.
A works as the right answer. If the confluence of cultures in the area contribute to conflicts over property, then it only stands to reason that as new cultures and peoples move into the region, property will continue to be fought over—and therefore potentially redistributed—in new ways. The only reason I might not choose A right away is because “will cause a redistribution of property” is a pretty strong claim; can I be sure that, on the basis of the passage, property will definitely be redistributed? Or can I only assume that there will be fights over property? That’s enough indecisiveness for me to hold onto A and see if I can eliminate the other answer choices before moving on.
The author spends the most time talking about how the conflicts encouraged by Western expansion are never over, but instead continually play out in the struggle between different communities. Since the author doesn’t describe an occasion for ending competition, B has to be wrong.
C also goes against the main idea of the passage. The lines on the nineteenth-century maps won’t be fragmented because the conflict described is not one in which smaller and smaller subdivisions are produced. Instead, property and the power of that property keeps shifting. Since the passage describes a very different kind of transformation than the passage does, C has to be incorrect.
There is no clear scenario in the passage in which anyone’s ethnicity no longer becomes relevant in their existence in the West. So D has to be wrong.
This question tests my grasp of the facts of Slocum’s act. I know that Slocum circumnavigated the globe by 1898, so this navigator would not challenge Slocum’s achievement of being the first person to circumnavigate the globe alone. So what else made Slocum’s achievement so notable? Paragraph 4 mentions that “Slocum had already been around the world five times on bigger ships,” and is warned by British naval officers that “A small ship sailed by a single man would not have a chance” against Mediterranean pirates. It sounds like the size of Slocum’s ship was a liability, and so a part of the challenge. And overcoming a challenge is what makes for an achievement. So I’m looking for an answer choice which mentions the size of Slocum’s ship, the Spray.
A works. If the Pearl was smaller than the Spray, then the navigator in the question stem would rob Slocum of the achievement of sailing the smallest vessel around the world. A is therefore correct.
B definitely suggests that the Pearl’s voyage was tough, but I can’t be sure how B would affect Slocum’s achievement without knowing how many times the Spray capsized. It’s not even clear if capsizing more or less is a greater achievement, since one can imagine that a ship which capsized less suggests the quality of the ship’s captain. B is therefore wrong.
C runs into a similar problem to B. The passage doesn’t tell me if the Spray was intercepted by pirates. Even if it was, it is not clear if getting intercepted by pirates makes the navigator in the question stem a better or worse sailor, or the achievement greater or worse. C is therefore incorrect.
D is tempting, because it would mean that the Pearl circumnavigated the globe more quickly than the Spray did. But the passage never suggests that part of the Pearl’s achievement was in its speed. If anything, the passage seems to celebrate how long the voyage took Slocum, since staying alone for that length of time was part of the challenge. So even though D suggests that the Pearl did something better than the Spray, that something is not part of Slocum’s achievement, so D cannot be correct.
The passage focuses heavily on Slocum’s isolation, especially when talking about how he would compare with other sailors today. This is clear in paragraph 5, when the author quotes Judith Lund: “Sailors these days often have autopilots, radios, phones, e-mail, computers, emergency locator beacons – even V.C.R.s for entertainment. It’s hard for us to imagine just how totally cut off Slocum was.” So I can predict that the right answer will mention that someone sailing alone today would be less subject to isolation than Slocum was.
Slocum’s funding is briefly mentioned in the passage, but the passage doesn’t clearly suggest that Slocum was poor. I’m also not quite sure whether it would be cheaper or more expensive for a solitary sailor to do what Slocum did now, so I have to eliminate A.
The passage does not suggest that pirate attacks have diminished now relative to Slocum’s time, so B is also incorrect.
C matches my prediction; I can choose it and move on.
D definitely has to be incorrect because, like B, the passage never suggests that the weather has gotten better since Slocum’s time.
There’s not much to be done about this question except to evaluate the answer choices and judge whether or not the passage helps me to understand the answer choice.
This is explained in paragraph 3: that despite Slocum’s historic achievement, “America was too busy becoming a world power to notice. Only a local reporter noted that Spray had come home.” So A has to be incorrect.
The passage explains how Slocum navigated in paragraph 2: “With only a compass, old charts, a sextant, and a hunk of lead on a line to measure depth, and a windup clock, Slocum had faced huge seas.” So B is also incorrect.
The passage mentions that people doubted Slocum’s achievement in paragraph 3, but “Slocum’s meticulous logs and Spray’s papers” all suggests that it was possible to know exactly when Slocum returned. So C is also wrong.
D is the last choice, and it is correct. The last paragraph states that Slocum only had a third grade education in the last paragraph, and so does not clarify how he appeared to be a “man of letters” to his audience. Since the passage does not clarify this, D has to be correct.
Why would Slocum’s journey have taken so much longer than anticipated? The most obvious reason is that Slocum had to turn around to circumnavigate the globe from a different direction at one point. This is stated in paragraph 4: “His plan had been to circle the globe west to east. But at Gibraltar, British naval officers warned him that the Mediterranean was swarming with pirates that season. A small ship sailed by a single man would not have a chance. So back he headed across the Atlantic. There were, he reasoned, two directions in which to sail around the world.” So I’m therefore looking for an answer choice which mentions Slocum having to double back in order to circumnavigate the globe.
A works. Slocum starts in America, crosses the Atlantic to reach Gibraltar, turns back and crosses the Atlantic again to circumnavigate the globe in the other direction, then crosses the Atlantic one more time to reach America. A definitely helps explain why Slocum’s journey would have taken so long, and aligns with the passage description of Slocum’s journey, so A has to be correct.
Paragraph 5 mentions that Slocum’s wife did not join him on his boat, so B can’t be right.
C goes against the last paragraph, which states that Slocum earned money by lecturing about his voyage, so it is unlikely that money issues would have slowed Slocum’s journey.
The passaged never talks about Slocum dealing with failed equipment, so D has to be incorrect.
The last paragraph talks about how Slocum made money by lecturing and appearing as an educated man. Since Thoreau is a famous writer, and since the paragraph is talking about his writing ability, the description is probably meant to suggest that Slocum was a very good writer.
I have no idea what Thoreau thought of the sea, so I can eliminate A.
I have no idea what Thoreau wrote about on the basis of the passage, so I can eliminate B.
I don’t know about Thoreau’s style on the basis of the passage, since the passage never mentions it, so I can eliminate C.
D comments on both Thoreau’s and Slocum’s writing ability, so D has to be right.
Given that this question is asking me for something that the passage does not say, the most efficient way for me to tackle this question is to search for the wrong answer choices: statements that are stated or implied by the passage.
The last paragraph tells me A is wrong. For the passage to say that “Market capitalism is based on greed—greed for profit and greed for consumption. The spiritual problem with greed is the delusion that it is the means to happiness.” To call greed a delusion is to imply that it is a false belief. That means that A is stated in the passage, and so has to be incorrect.
The passage does not argue this. Greed is built into capitalism, but the passage never claims that it is inherent in human nature. If B were true, then the author would not have reason to hope that we can find ways of relating to the world that are not so dominated by capitalism (last paragraph). Since B is neither stated nor suggested in the passage, B must be correct.
This is stated in paragraph 4: “The current economic system reflects the ascendancy of a particular way of understanding the world, and its creed need not be accepted. Its ontology and ethics compete with other understandings of the world and the way that one should live in it.” To say that our economic system says something about how we should live in the world is to say that economics has some role to play in our social arrangements: how we are arranged in society. C is therefore incorrect.
D is a bit of a confusing answer choice. But to say that “Market capitalism is only one worldview” is to say that it is one worldview among many. This is stated at the end of the passage: “Fortunately, other worldviews exist that can still help us to do so.” That means that D is implied by the passage, which makes D incorrect.
Another question where I cannot predict what the right answer will look like. Instead, I need to discern which answer choice best describes the passage and the author. I know that the author is clearly concerned with capitalism and greed, to the point of arguing that we should find a different economic system. That should hopefully be enough for me to find a right answer.
While the passage describes economics as a religion, the author is not clearly arguing for a religious solution. That alone tells me that A has to be incorrect.
B makes sense as the right answer. The passage clearly protests against capitalism and our current economic system, and it would be fair to call the author’s approach philosophical, since it deals with the problems of market capitalism on the level of concepts and ideas. I would therefore choose B and move on.
I can see how the passage could be described as grim, but the passage only briefly touches on the social implications of economics. For C to be correct, I would need to see more about society and economics than the passage already deals with. That means C is incorrect.
I can also see how one could call the passage well-reasoned, but while the passage talks about economics, it does not do so in a way that an economist would. I also know D has to be incorrect because the author does not suggest an economic solution.
The relationship between science and religion is mentioned in the first paragraph: “If instead we adopt a functionalist approach and recognize that our religion is whatever system enables us to understand the world and our role in it, then it becomes obvious that the traditional religions are fulfilling their functions less and less; they are being supplanted by alternative systems. Today the most powerful explanatory system is science, and the most attractive value system is consumerism. The academic offspring of these two orientations is economics, the most influential of the social sciences.” If religion is an explanatory system for reality, as is science, then I’m looking for an answer choice which suggests that one can take the place of the other, since they fulfill the same function.
The passage does not suggest that they are mutually exclusive. I also know that the passage says that religion and science are explanatory systems; the only value system in that first paragraph is consumerism. So A has to be wrong.
I don’t see anything in the passage about different levels of reality, so B has to be wrong.
I also don’t see anything in that first paragraph that explains how science and religion can be compatible, so C has to be wrong.
D is the last answer, and it matches my prediction, since it states that both fulfill the same function: offering views of reality. D is therefore correct.
Religion is described in the first paragraph, but that discussion doesn’t really seem to help me. More importantly, the passage isn’t really about religion per se, but rather economics as religion, which is mentioned in paragraph 1 and 2. I know that the author believes that economics as a religion is a problem: it lies about being able to make people happy (last paragraph). That sounds a lot like being dissatisfied, so I’m looking for an answer choice that mentions people accepting the religion of capitalist economics, and finding that to be unsatisfying.
The passage isn’t really about religion, so A has to be wrong.
B gets a bit closer, because it echoes paragraph 1, which mentions that traditional religions are failing. But the specific reason they are failing has nothing to do with their ability to make people happy; instead, paragraph 1 claims that traditional religions are failing to fulfill their function as explainers of the world. Since the passage never says B, B cannot be correct.
C is most in line with the passage’s main idea: that consumerism is a religion that cannot satisfy people. I would therefore choose C confidently and move on.
I don’t know anything about the effects of being committed to the values of a society based on the passage, so I can eliminate D.
I know that the passage defines religion in the first paragraph: “our religion is whatever system enables us to understand the world and our role in it.” I also know that the author is concerned with how market capitalism has become a religion (paragraph 2). Since the author states this concern most clearly in paragraph 4—“The current economic system reflects the ascendancy of a particular way of understanding and valuing the world,”—I am looking for an answer choice that either claims that market capitalism helps us to understand our world better, or that teaches us ways to value the world.
A matches my second prediction: that market capitalism teaches us ways of valuing the world, since that implies, by definition, a system of values. A has to be correct.
I can disregard B because the passage never claims that capitalism unites us: that is, is a force that is intended to unite. The claim that market capitalism is “becoming the first truly world religion” (paragraph 2) is less a matter of uniting and more a matter of conquering, which is a slightly different concept when it comes to religion.
The passage is pretty concerned about capitalism, so there’s no clear way that the author would suggest that it is defensible as just and right.
D is wrong because it claims that market capitalism “transcends” happiness, when it only promises happiness (last paragraph). Transcending is a very specific term that requires specific arguments to suggest it.
I know utilitarianism is discussed in paragraph 5: “According to utilitarianism, society is composed of autonomous individuals, each seeking personal ends. Human values are reduced to a calculus that maximizes pleasures (without distinguishing qualitatively among them) and minimizes discomfort.” So I’m looking for an answer choice which describes someone using unexpected money for their personal pleasure, or to minimize some personal discomfort.
A requires that I add an assumption to be correct: that the utilitarian would derive some happiness from owning what other people value. Anytime an assumption is needed to make an answer choice look right, it is a good indication that the answer choice is wrong, so I can eliminate A.
B describes a necessity and not something which is meant to increase pleasure or minimize the discomfort of the autonomous individual. B would be more attractive if it was the utilitarian who needed the medical care. B is therefore incorrect.
C is far too altruistic to fit the definition of utilitarianism in the passage, which is focused on selfishness: on what benefits the individual. That makes C incorrect.
D is the last answer, so it has to be right. Travel especially feels like a clear indication that this is the right answer, since people tend to travel for pleasure. Most importantly, D describes satisfying desires, which is another way of describing pleasure.
I can answer this question by going to the same part of the passage I went to for the last question: the passage discussion of utilitarianism. I know this because market capitalism is the reigning economic theory, and because contemporary economic theory is “in thrall to utilitarian values.” So I can be sure that the market capitalist would not respond to the proposal by accepting it on the basis of its humanitarian value; the capitalist would want to consider personal gain first and foremost. I also know that capitalism is equated with greed (last paragraph), so I would also imagine that the right answer would have to mention profitability.
There is nothing clearly market capitalist about A, since it lacks the distinguishing aspects of the market capitalist: selfishness and greed.
The passage does not suggest that the market capitalist would be worried about morality. Given the author’s disdain for market capitalism, it is easy to imagine the author suggesting the opposite: that the market capitalist would want to know how to profit from medical emergencies.
C matches my prediction, since it is both selfish—as financial returns on investments can be—and focused on profitability. I can choose C and move on.
D is what I predicted the answer would definitely not be. The author is very concerned that market capitalism lacks the sense of humanity that D would require. That makes D incorrect.
The phrase “viewed with suspicion” tells me I need to go to the first paragraph, which says that “In an aggressively egalitarian society, culture has always been a suspect word, suggesting the pretensions of an effete and foolish leisure class.” So I’m looking for an answer choice which mentions that culture seems to connote a way of thinking associated with an upper class (the only class that could be considered both foolish and a leisure class).
The author mentions this conflict later on, but that conflict is the result of the loss of a shared culture; it does not have anything to do with culture itself. I can therefore eliminate A.
This is the author’s definition of culture, but it is not why the author says some people are suspicious of culture. B is therefore wrong.
C matches my prediction: that culture is associated with an elite group (an effete and foolish leisure class). C is definitely correct.
D is another way the author defines culture in the first paragraph. But, just like B, D cannot be correct because the author’s definition of culture is not given as a reason why people are suspicious of culture.
A quick glance at the answer choices tells me that the right answer will require me to discern how the author would judge a television version of The Scarlet Letter and why. The Scarlet Letter comes up in the last paragraph, where the author, after having mentioned that some movies may take the place of books as cultural touchstones, explains that most movies and television shows would not: because they are nowhere near as long-lived as the great novels are. The author favors The Scarlet Letter because “The symbolism of Moby Dick and The Scarlet Letter, however much one resented being force-fed them in school, is something everybody can be expected to share.” So while the author would likely be wary of any television series, the previous paragraph suggests that the author is also willing to admit that newer forms of popular media could be worth watching from a cultural standpoint. So I can guess that the author would suggest that a television version of The Scarlet Letter would be worth watching, because hopefully something of the book would survive its transcription into another form of media.
A does not match my prediction because the author likes The Scarlet Letter enough to probably think that a television version of it would be worth watching. So I can eliminate A.
The author is pretty clearly pro-book, and I don’t have anything in the passage that would suggest that there is some case where a television series could be better than a novel, so I can safely eliminate B.
C feels like a safe answer choice here. Since the television movie or series in question is going to be based on a book the author thinks is culturally important, and since the author is clearly down on most made-for-television movies, I can justifiably assume that the author would like a television movie on The Scarlet Letter better than those made-for-television movies that the author thinks lack cultural content. C is therefore a good choice for the right answer.
I don’t have a good reason to choose D. On the other hand, the author definitely thinks that classic films are worth watching, since they have become cultural artifacts, and the author clearly thinks that television is not yet a cultural resource because nothing on television persists (last paragraph), so I have a good reason to eliminate D.
Though this sounds like a general question, the author spends most of the passage lamenting how much U.S. culture seems to be diminishing. The only clear pieces of media that the author claims are part of America’s shared culture are the great books like Moby Dick and The Scarlet Letter (last paragraph) and films, since the author says in paragraph 5: “Films, to be sure, have entered into the U.S. cultural identity in an extremely powerful way.” So the right answer will likely mention movies.
A mentions a movie, and so I would choose it and move on.
Music is not mentioned in the passage, so B has to be worse than A.
The author clearly hates television in the last paragraph, so there’s no way that C could be correct.
D mixes two forms of media here: film (since Brando is mentioned in paragraph 5 as a film actor) and books. But that mixture is what makes D wrong. The author’s point is that Brando would be part of a cultural artifact in a movie, and that there are some great books that are part of U.S. culture. But that does not mean that a book about an actor would be a cultural artifact.
To answer this question, I need to figure out what part of the passage the information in the question stem affects. Education is mentioned in paragraph 4, where the author laments that reading is taught as a “means of survival, not as a source of pleasure and of shared experience.” The next sentence, which mentions classics, is more to the point: “The notion of ‘great books’ is viewed by most educators as an archaic concept, relegated to the museum of old teaching devices.” I can imagine, then, that the teaching of mystery novels would come off to the author as proof that educators view the idea of the “great book” as an outdated concept.
A is almost an exact match for the part of the passage I predicted would contain the right answer, so I can choose A and move on.
The passage only tells me that television is a more accessible media than books. I don’t really have a good reason to think that mystery novels are more accessible than the classics on the basis of the passage. B would only be attractive if I brought in outside information or my personal biases into the passage, which of course I should never do.
C is from the right paragraph, but there is no clear way to discern how teaching mystery novels is a way to teach reading as a survival skill. So C has to be wrong.
D can’t be right because it’s making a claim that the information in the question stem cannot support. I only know that some English courses include mystery novels and perhaps avoid the classics; for D to be correct, I would, at the very least, need to know that most—the majority of—English classes avoid the classics.
The author’s mention of Moby Dick takes place in the last paragraph, where I know the author expresses skepticism over the possibility of viewing television as a form of culture. There, the author says “The symbolism of Moby Dick and The Scarlet Letter, however much one resented being force-fed them in school, is something everybody can be expected to share.” I can deduce two claims about Moby Dick from that: that everyone has read it and so it is a source of culture, and that some people were forced to read it in school.
A almost matches my second prediction. However, A has to be wrong because it plays with percentages. The passage only says that some people were force-fed books like Moby Dick in school, and that what Moby Dick provides is something that could be shared by all. That’s not enough for me to know that most people were force-fed Moby Dick in school: that is to say, over 50% of people were.
This is the author’s opinion, but it is not clearly one that is attributed to the vast majority of people in the last paragraph. So B has to be incorrect.
There is nothing in the passage that clearly suggests C. I know that people are forced to read the “great books,” and I know that “many millions of people” watch television (last paragraph), but those are both so inexact that I can’t be sure that one is greater than the other. That makes C incorrect.
D matches my prediction that Moby Dick is a source of culture. D also must be right because the entirety of the last paragraph is dedicated to arguing that there is little to no potential for shared culture in television series.
The mention of society and culture brings me back to the passage’s definition of culture in paragraph 1: “culture actually refers to nothing more objectionable than a system of shared symbols and examples that hold a society together.” So I’m looking for an answer choice which brings up this idea that culture is what keeps a society together.
Economic problems are not mentioned in that definition of culture from paragraph 1, so I can eliminate A.
B feels like it fits. If culture holds a society together, then a society which lacks a culture is one that lacks something that helps keep that society together, which in turn suggests that that culture will have a harder time coming together and staying together. I can choose B and move on.
The author argues the opposite in the last paragraph: that television programming seems to be without cultural content, and so cannot impart cultural identity. That’s enough to make C incorrect.
This is from the same paragraph as the definition of culture, but it is posed in the first paragraph as an opinion held by a few: just those “In an aggressively egalitarian society.” D is also wrong because it’s too narrow; the question is asking about culture, while D only references a society’s “great books,” which are just an aspect of a society’s culture.
This is a main idea question, since the author’s argument centers on the idea that tragedies require their heroes to be flawed or to make flawed judgments (paragraph 1). By paragraph 3, the author claims that this understanding is flawed: “The human flaw does not always exist in tragedy—and tragedy cannot be defined or explained by it.” The author then goes on to explain how we may have come to view the flaw as necessary for tragedy, but the passage ends with “the flaw is a concept that becomes maddeningly elusive under the most cursory examination of actual plays.” So the author’s argument is that the fatal character flaw is not a sufficient definition of what makes a play a tragedy: that tragedies are more complex than just plays which contain flawed characters.
A is a test of how well I understand the author’s main point. While the author expresses some reservations about the tragic flaw, the author never goes as far as A suggests. The author admits that some tragic heroes have flaws (paragraph 3), so fatal character flaws cannot be irrelevant to the downfall of tragic heroes.
B matches my prediction. To say that the character flaw is too facile a way of interpreting tragedies is to say that it is too simplistic to reduce tragedies down to fatal character flaws. B is therefore correct.
C speaks to the heart of the passage. The author argues that actual tragedies do not necessarily use character flaws: that it is therefore not a standard plot device in classical dramas. We only think that is the case because, the author argues, we have generated the expectation that tragedies have to have flaws. C is therefore incorrect.
D is exactly what the author argues against, so it has to be wrong.
Aristotle’s arguments about tragedy come up in paragraph 5, where the author claims that “We are bound in reverence to attend to Aristotle; he is the only philosopher-critic to have written extensively about tragedy while tragedy was first in the process of defining itself.” The author’s use of the present tense here tells me that the author is explaining why Aristotle remains influential into the present: into the now of the passage. So I’m looking for an answer choice which says something about Aristotle writing about tragedy as tragedy was first being formed.
A is a really attractive answer choice because Paragraph 6 mentions that “drama after the Renaissance does display a flawed protagonist more regularly and more explicitly than Greek drama ever did.” But A has to be wrong because the author does not suggest that A is the reason why Aristotle’s writings on tragedy remained influential. The author actually suggests that the causal relationship here is the opposite of A: that the reason why the work of later playwrights so resembles Aristotle’s writings is because those writings remained influential.
B matches my prediction, and so I can choose it and move on.
The author lists this as part of the logic of seeing tragedy as the result of some tragic flaw, but this reason is not given by Aristotle anywhere in the passage. So it is difficult to see how C could possibly be the right answer.
I don’t see anything about Aristotle defining tragedy in an understandable way, so I can safely eliminate D.
This is a flaw question; it is asking me to identify some weakness in the passage argument. Arguments like this one, which argue that something should not be defined in a certain way, are reliably flawed insofar as they often fail to consider alternative explanations: fail to explain, in the case of the passage, how we should define tragedy. But flaws are difficult to predict, since these passages are written by professional writers who are unlikely to leave really obvious flaws in their arguments, so my best strategy is to evaluate each answer choice and ask if 1) it describes the passage accurately, and 2) if A would weaken the passage argument.
A fulfills both criteria. It accurately describes the passage argument—there are indeed no examples of tragedies with unflawed characters in the passage—and A would hurt the passage argument by suggesting that the author’s point lacks real-world evidence to support it. A therefore has to be right.
The author definitely does do this, so criteria 1 is fulfilled, but I’m not sure how B weakens the passage argument. The author is making a point about how some people define tragedy; I could imagine the author leaving out all the parts of the argument that mention reader beliefs, and the passage would still make sense. So since B does not clearly weaken the passage argument, I can eliminate it.
C refers to the passage argument insofar as the author claims that people believe tragic dramas always punish misdeeds, so criteria 1 is fulfilled. But even if C was true, it would not clearly weaken the passage argument. Even if every tragic drama contained a misdeed that was punished, that does not necessarily suggest that we should define tragic dramas on the basis of whether or not they contain misdeeds that are punished. The author’s point is that the tragic flaw makes less sense the more you look at it; having misdeeds punished all the time, as people expect them to be now, doesn’t clearly clarify the tragic flaw, so C has to be wrong.
D is wrong because it fails criteria 1. The author never suggests that literary forms don’t have features in common. The author is only talking about a single literary form—the drama—so D is actually unrelated to the passage argument.
I know that I have to figure out how the situation in the question stem relates to the passage argument. But it’s difficult because the passage is all about tragedy, and the film mentioned is likely to be a comedy (since it contains a comedian, Buster Keaton). It’s pretty clear that the passage is suggesting that Buster Keaton’s character is making a mistake in addressing one leak by making another. I suppose that means that Buster Keaton’s character is a flawed character, but one that is not in a tragedy. That brings to mind the passage claim that “The human flaw does not always exist in tragedy—and tragedy cannot be defined or explained by it” (paragraph 3), since here we have a human flaw that is not in a tragedy.
A would require buying into the logic that the author opposes. The quote from paragraph 3 suggests that the author does not think that the presence of a flaw makes something a tragedy, so A has to be wrong.
B runs into the same problem: that I can’t derive a definition of tragedy from the comedy described in the question stem. So B is also wrong.
Nothing in the passage suggests that characters who have no choices are tragic. The passage actually seems to imply the opposite: that the popular view of tragedy implies that people make the wrong choices, which is only possible if people have choices to make. C therefore has to be incorrect.
D matches my prediction, and the logic in the quote from paragraph 3, so it has to be correct.
Saints are mentioned in paragraph 3, as a term applied to some “tragic characters who seem altogether unstained but suffer nonetheless.” But saints are raised as part of the argument in that paragraph: that “The human flaw does not always exist in tragedy—and tragedy cannot be defined or explained by it.” In other words, the author is saying that tragic flaws—the nature of tragic characters—should not be used to explain their ultimate tragic ends.
This is incorrect: both because it does not match my prediction, and because the author is focused on what makes characters tragic: not why they act the way they do.
The passage doesn’t tell me anything about how characters respond to various situations. The passage is focused on arguing against a perceived commonality in tragedies: the idea that tragic characters cause their own tragedies through the presence of some flaw. So B can’t be correct.
C matches my prediction. Saints are examples of people who go through a tragedy, but are pure and good, and so lack a tragic flaw. It would make sense to say that saints are therefore examples of people whose nature does not predict their tragic downfalls.
D is very similar to A, since both talk about motivations. D is thus wrong for the same reason A is wrong: because the author is not interested in the motivations of tragic characters.
To answer this question, I need to judge the passage tone. The author is very confident in the point being argued; so much so that the author is willing to both explain what makes the point the author is arguing against so attractive (paragraphs 5 and 6). Hopefully that is enough for me to find a correct answer.
I wouldn’t say the author is questioning; to say that is to imply that the author does not come up with a point to argue. The author is clearly challenging a conception of tragedy, so A has to be wrong.
Assertive here feels pretty good, since the author is confident in the point being argued, but I’m not terribly sure about whether or not the author is “earnest.” I’m not sure how to judge whether that is true or not; the author isn’t being sarcastic or playful, but that doesn’t seem like enough for me to confirm B. So I’m going to hold onto B and see if I can eliminate C and D.
“Confiding” implies that there is some secret to be confided. But there is nothing secret in the passage, so C has to be wrong.
The author isn’t really accusing anyone of anything. If D were correct, I would expect the author to be a lot angrier at Aristotle—since he first comes up with the notion of the tragic flaw—or at the Renaissance playwrights who helped cement the notion that tragedy = tragic flaw. But the author doesn’t seem particularly angry or mean at either of them; instead, the author seems somewhat understanding. So since I can’t see how the passage is accusatory or even stern, D has to be wrong, and B correct.
To answer this question, I need to be sure that I know Aristotle’s claims. These are given in paragraph 5: “The figure who serves the tragic plot most satisfactorily, he tells us, is one whose misfortune is brought about not by consistent vice or willfulness but by some error in judgment.” So I imagine that Aristotle would warn students to avoid errors in judgment. The passage goes on to clarify Aristotle’s claim, though, at the end of paragraph 5: “Aristotle really meant an aberration brought about by one or another form of self-indulgence.” So I should also be on the lookout for an answer choice warning people to not be selfish.
A works: both because failing to read a test question carefully is an error in judgment, and because it is a form of self-indulgence: that is, laziness or haste. Still, this is an analogy question of sorts—asking me to analogize Aristotle’s argument about tragedy into a situation involving students—and analogy questions are hard, so I should eliminate the other answer choices before moving on.
B advocates regularity, while Aristotle claims that tragedy comes about “not by consistent vice or willfulness,” so B has to be wrong.
An attitude is also regular, so C is wrong for the same reasons B is wrong: because Aristotle wouldn’t warn people to avoid being consistent, since it is only inconsistent actions that can be tragic.
I don’t see an error in judgment in D. To say something is compulsive is to say that it is too instinctual to be an error in judgment, so D is not something that Aristotle would warn a student about.
This is an odd question because it is one that refers to the readers of the passage. But the author does refer to the passage’s readers in paragraph 6: “The presumed pattern is reinforced by our tendency to think of tragedy as moral illustration.” The pattern referenced is one in which flawed protagonists cause their own tragedies. So the quote from paragraph 6 tells me that readers like me—“our tendency”—equate tragedy with moral illustration: that is, that evil actions get punished, and good actions get rewarded.
A matches my prediction about how bad deeds get punished, so A has to be right.
The passage discussion of what readers of the passage think focuses, unsurprisingly, on what readers of the passage think: not on what playwrights believe. This one difference from A is what makes B worse than A.
C goes further than the passage does in describing the common conception of tragic characters. It clearly breaks with Aristotle’s definition—the definition that our common conception of tragedy comes from—since someone who fits the description of C would be too consistently evil to fit a tragic plot. That’s enough to make C wrong.
The passage never says that the gods choose victims randomly. I also don’t see how D relates to how readers like me view tragic characters. That makes D out of scope enough to be incorrect.
I know that the author brings up how much trash is around the hut, but the author seems to focus on how locked in the hut is: “inside the fence, which I peered through like a prisoner, was the hut, a forlorn relic overpowered by what had been done to protect it from collectors of mementos […] There it was, perhaps a vital clue to our history and our inheritance, turned into a curio” (paragraph 4). Since this is an analogy question, I’m looking for an answer choice that describes something that is similarly locked behind a gate, or rendered inaccessible. Or else I’m looking for trash that is left on the way to something important.
There is nothing closed off about A—if those frontier towns are truly abandoned, then there would be no one to maintain barriers around the collapsing houses—and there is also no clear analogue to the trash around the hut in A, so I can eliminate it.
B has to be wrong, even though one might think of graffiti as similar to the trash around the hut, because subway tunnels lack the sense of historical importance that the author ascribes to the hut.
C could be correct. Glaciers are considered to be important, and the equipment left on them could be read as trash. I’m not especially sure of C, though—since glaciers might not be historically important in the way that the hut is—so I’ll hold onto C, but check and see if D is incorrect.
While D mentions rubble, that is not clearly analogous to the trash the author describes surrounds the hut. That trash is not valuable, while the pottery shards are arguably valuable in helping people to understand how prehistoric people lived. Since D does not resemble the passage description about the hut, C has to be correct.
I know that the author is upset at the negative effect of the hut’s popularity. So if the Gorge became like a busy street, the author would likely think that is a bad thing. Looking at paragraph 6 confirms this, since the mention of the busy street is preceded by the author saying that “I consider the ironic possibility that what I will write for love of the Gorge may also contribute to its destruction, enlarging the hearsay of it, bringing in more people to drive the roads and crowd the ‘points of interest.’” So I’m looking for an answer which mentions that the Gorge will become too popular: too crowded with people.
A is definitely one way to say that the Gorge would have too many visitors, so A has to be correct.
For B to be correct, the author would have to suggest that the hut wasn’t interesting. But the author is clearly interested in the hut—because Daniel Boone might have been there—so B has to be incorrect.
Technological advance is not a major topic in the passage, so I know C has to be wrong.
For D to be correct, the passage would have to suggest that busy streets were streets without trash in them. But the passage leaves the discussion of trash a few paragraphs back and does not mention it again, so D can’t be correct.
The discussion of being a prisoner stands out in my mind because it is implied in one of the most beautifully written moments in the passage: “When people are so ignorant and destructive that they must be divided by a fence from what is vital to them, whether it is their history or their world, they are imprisoned” (paragraph 5). I know that the author feels “like a prisoner” upon seeing the hut sectioned off by fencing. So I can predict that the image of being a prisoner is meant to imply that one is cut off from something important: from something vital.
The author here is describing just the hut, and not the Gorge as a whole, so A can be eliminated for going too far.
The author is specifically referring to the hut, and not the earth as a whole. B therefore has the same problem as A does, and so is wrong.
C is a really tempting answer choice, because it does seem to describe the hut being locked behind a fence. However, I know C has to be wrong because the passage describes the author, and not the hut, as the prisoner (paragraph 4).
D is the last answer, and luckily it almost perfectly matches the part of paragraph 5 I quoted. D is therefore correct.
Though the author begins by talking about the hut because of its possible connection to Daniel Boone, very little of the passage actually involves the hut’s connection to Daniel Boone. The passage is more about how humans damage and cut themselves off from what matters to them. The author could made the same argument about any other well-visited site in the world. So I’m going to guess that if the hut actually had no connection to Daniel Boone, the author’s argument would be unaffected.
A matches my prediction, which is particular enough that I can choose A confidently and move on.
B sounds great because the author is clearly unhappy with tourist attractions in paragraph 6. But in that same paragraph, the author makes it clear that the passage is making a claim about more than the Gorge: “I might as well leave the place anonymous, for what I have learned here could be learned from any woods and any free-running river.” That means that the author would not likely react as B describes, which makes B incorrect.
C has to be wrong because so little of the passage actually talks about the long hunters.
D is wrong because the damage to the site is symptomatic of the author’s larger concerns about how people can hurt the natural world while loving it. The author would therefore not focus on the trash itself, but what the trash signifies. That’s enough of a difference in focus to make D incorrect.
Paragraph 4 only directly references the hut’s visitors at the end of the paragraph: “Whether because of the ignorant enthusiasm of souvenir hunters or because of the strenuous measures necessary to protect it from them, Boone’s hut had become a doodad.” There’s a clear sense of annoyance with these visitors who are so enthusiastic that the site they came to visit needs to be protected from them. That should be enough for an answer choice.
Bemusement implies that the author derives some pleasure from these tourists. But there is no clear pleasure here; the author is pretty negative about them, so A can’t be correct.
Disdainful has the right tone, since it is a negative word. However, it is possible that another answer choice might also have a negative word, so before I choose B, I should check the others.
Tolerant is a positive term, while the author clearly dislikes those who visit the hut, so C has to be wrong.
The author clearly cares about the hut and has a strong opinion about those who visit it, so apathetic doesn’t feel like an accurate description of the author’s perspective either. D has to be incorrect, and B correct.
This is a main idea question: one asking if I can describe the passage in generalized terms. I know that the passage begins with a history of the Social Security system, and then shifts into a discussion of why people seem willing to keep raising taxes instead of reducing benefits. This ends up amounting to something of a defense of the Social Security system, especially because the passage ends with a startling statistic: “only 15 percent of those surveyed favored changing the law to eliminate the tax on individual workers.” So I’m looking for an answer choice which seems to defend Social Security because people seem to be highly in favor of it.
A is a defense of Social Security, but “fairness” doesn’t really seem to factor into the system’s popularity. Paragraph 4 actually suggests that Social Security might not be fair, since payroll deductions are described in that paragraph as “regressive taxes”: taxes, that is, which tend to burden one less the more one makes. Regressive taxes are therefore most onerous on the poor. So A has to be wrong.
B is not a bad answer choice, because it describes the passage as a general defense of Social Security: one based on the satisfaction, or popularity, of the program. Still, I don’t remember any actual allegations of growing dissatisfaction with social security, so I will hold onto B and just make sure C and D are incorrect.
The passage never suggests that there isn’t a need for Social Security, meaning that the passage does not have anyone to attack. C is therefore wrong.
Roosevelt is mentioned in the passage, and is described as a “prophetic character,” but D has to be wrong because Roosevelt plays such a small part in the passage argument. He only appears in one paragraph in order to explain why people are so opposed to reducing benefits, and so willing to raise taxes. Since Roosevelt is used to make a different point, it wouldn’t make sense for Roosevelt to be the focus of the passage. That makes D wrong and B correct.
This is a tough question to predict the answer for, since the entire passage is about Social Security, and most of it is about Social Security taxes. So my best strategy here will be to just go through and test each answer choice: eliminating choices that go against the passage argument.
A works. I know from my work on one of the wrong answers in the previous question that the author mentions how regressive payroll deductions are. This claim is made in paragraph 4, as an implied reason for Roosevelt to not create the Social Security program. Because neither Roosevelt nor the author directly contest A—Roosevelt actually acknowledges that payroll deductions would likely be a regressive tax—I can assume that the author would agree that Social Security is actually regressive. I can choose A and move on.
This directly contradicts paragraph 5: “Social Security was never welfare.” B therefore has to be wrong.
While the author does bring up a problem that Social Security is facing in paragraph 2—“Social Security taxes have risen dramatically because benefit levels have been raised and the proportion of the employed paying into the system has declined relative to the proportion drawing benefits”— the author’s mention of tax hikes to cover the increased cost of benefits is actually a sign that Social Security is currently economically sound. C therefore has to be incorrect.
There is nothing in the passage which suggests I should choose D. Younger workers aren’t really talked about much in the passage. The passage also does not talk much about people who dislike Social Security. Without more information, D has to be wrong.
The word “welfare” brings me to paragraph 5: “Social Security was never welfare. Its benefits were to be bought and paid for by individuals to provide for their economic security and well-being.” The author crystallizes this point later in the paragraph: “Workers wanted to ‘pay their share.’” So I’m looking for an answer which suggests that every worker was paying for their own benefits.
A works. It is almost an exact copy of the quote from paragraph 5.
Need is, surprisingly, not discussed much in the passage. It is implicitly raised when the author considers how regressive payroll deductions are, but that is not tied to the perception of Social Security as a non-welfare program. B is therefore wrong.
C is contradicted by the last two paragraphs, which describe a survey asking if people wanted employers, and not employees, to pay for Social Security taxes. The fact that the vast majority of people voted “no” means that employer contributions do not likely play a role in how people see Social Security.
I don’t remember any mention of replacing lost income, so D has to be wrong.
This is a very prominent argument in the passage, if only because it is so surprising. I know that the author brings up how people are willing to pay more in taxes in order to avoid decreases in benefits (paragraph 3), so the right answer has to say something to that effect.
A is attractive because it implies a resistance to cutting benefits in the face of cost-of-living increases. I can confirm A by looking at paragraph 3, which mentions such adjustments: “If a choice must be made, raise taxes rather than cut benefits. More recently, respondents by a roughly three-to-one margin have expressed their opposition to limiting cost-of-living adjustments for Social Security recipients in order to reduce the federal budget deficit.” If people oppose limiting cost-of-living adjustments, then they must be in favor of maintaining those adjustments. A has to be right.
Paragraph 3 says the exact opposite, so B has to be wrong.
C contradicts the last two paragraphs, where a Gallup survey that asked if people wanted employers to pay for all Social Security taxes received an overwhelming number of “no” responses. C therefore has to be wrong.
There is no discussion of private replacements for social security in the passage, so D has to be wrong too.
The passage only raises two reasons to be worried about public support for Social Security, and both are given in paragraph 2. The first is that “Social Security taxes have risen dramatically,” and the second is the reason why: “benefit levels have been raised and the proportion of the employed paying into the system has declined relative to the proportion drawing benefits.” So the right answer will either mention the higher Social Security taxes, or the relative decrease in the number of payees into the system.
It’s not clear if A is true or not. The passage mentions that benefit levels have been raised, but that could either mean that the real value of those benefits has stayed the same as they were in the 1930s, exceeded those values, or is less than those values. Without more specific data or a more specific argument about the real value of Social Security benefits, I don’t have a good reason to choose A.
B sounds right, and if I’m in a rush, there’s a chance that I would choose B because it sounds like what I’m looking for: a choice that says something about raised taxes. But B is phrased in a way that says more than the passage actually suggests. I know that taxes for the program have been rising, as have been benefits, but I have no idea how the two rates relate to each other. Since the passage doesn’t give me that information, B has to be wrong.
C is the opposite of what’s occurring. The problem Social Security is facing is a decreasing number of workers supporting an increasing number of beneficiaries. C is therefore also wrong.
D is the opposite of C, and is therefore correct.
The long first paragraph makes figuring out the main idea more challenging than it might otherwise be. I know that the passage begins by talking about how Victorian female authors had to hide their passions in order to succeed in the literary world. The second paragraph supports this claim and then builds on it: “It does sometimes seem as if feminine writers are metaphorically paralyzed by refinement and restraint, but the repression in which the feminine novel was situation also forced women to find innovative and covert ways to dramatize the inner life, and led to a fiction that was intense, compact, symbolic, and profound.” In other words, the constraints on female writers did encourage innovations in novels by female writers. Since the last paragraph just lists one of those innovations—the man who came to represent the projected ambitions of female novelists—the right answer has to mention something about either how constrained female writers were, or the benefits of that constraint.
The author definitely laments how the English reading audience seemed to reward bland prose, but this claim is part of a larger argument about how women writers were constrained. Since the passage uses A to make a different point, A cannot be the central thesis of the passage.
The last paragraph mentions that “Many of the fantasies of feminine novels are related to money, mobility, and power,” and that those novels often played out those fantasies through their male characters, but the passage never states that feminine novels were “concerned mainly” with those issues: that is to say that a majority of feminine novels held that concern. Since B distorts the passage by going too far, it has to be incorrect.
C matches my first prediction: that the passage argues that female writers faced constraints that affected their writing. I can therefore choose C and move on.
The passage actually suggests the opposite in paragraph 1: that for female authors, “a bland and gelatinous prose won applause,” not an innovative prose that was full of passion and reference to the inner lives of their writers. D has to be wrong.
In the last paragraph, the author claims that men in some novels served as a vessel for the ambitions that women were not supposed to have or display. The list of what those ambitions are come up right at the beginning of the last paragraph: “Many of the fantasies of feminine novels are related to money, mobility, and power.” So the right options will likely refer to these.I. While ambition and assertiveness are not listed in that first sentence, they do make it into the second. For the author to mention that “feminine novelists punished assertive heroines” but then “dealt with personal ambition by projecting the ideology of success onto male characters,” the author is suggesting that those feminine novelists wanted to portray assertiveness and ambition, but did not feel like the literary world would allow those qualities to be attributed to a woman. So, instead, those writers attributed those feelings to men. That makes I correct.

II. II is right for the same reason: because I can read the qualities given to male characters in feminine novels as qualities the female author’s possessed but could not portray as belonging to women. The last paragraph states that male characters had “initiative, thrift, industry, and perseverance.” Since II does as well, II has to be right.

III. III is a test to see if I’m paying attention to the question stem. I know that the passage is about repression, and the depiction of some men in feminine novels are a fantasy, but that does not mean that female authors had fantasy and repression as personality traits. That doesn’t even necessarily make sense to say, so III has to be wrong.

III is definitely wrong, so A is wrong.
B matches my prediction—that I and II are correct—so B is correct.
C is wrong because III is wrong.
And D is wrong because III is wrong.
This is the primary subject of the first paragraph, so it is likely that there will be many correct options from throughout that paragraph. The pertinent sentence from the first paragraph is: “The verbal inhibitions that were part of the upbringing of a lady were reinforced by the critics’ vigilance.” Of note here is obviously the mention of critics, but also that the critics only reinforced something that was part of the “upbringing of a lady,” or something that was part of their education.I. This matches the reference to a lady’s “upbringing,” so I has to be correct.

II. Critics are also mentioned in that sentence as encouraging verbal inhibitions, so II has to be correct.

III. III is not mentioned in that specific sentence, but III is implied by the start of the paragraph: “The training of Victorian girls in repression, concealment, and self-censorship was deeply inhibiting.” So III has to be correct as well.

I need all three, so A is incorrect.
Same with B.
Same with C.
D has all three, so it must be correct.
The only moment of praise in the passage stands out because of how much the passage focuses on the ways the literary world kept women down. It is towards the end of paragraph 1: “While verbal force, wit, and originality in women were criticized, a bland and gelatinous prose won applause.” So I’m looking for an answer choice which mentions how women won applause for having a boring writing style.
A is an exact match for my prediction, so I can choose A confidently.
The middle of paragraph 1 suggests the opposite: that Victorian readers would “rebuke unconventional language in women’s literature.” So I can eliminate B.
This is something that the author respects in women’s literature of the time, but it is not something that is clearly attributed to the praise of critics, so C is wrong.
This is contradicted in the first paragraph: “Reduced to a pastoral flatness, deprived of a language in which to describe their bodies or the events of their bodies, denied the expression of pain as well as the expression of pleasure, women writers appeared deficient in passion.” Since women could not express pain, they would not be praised for such expressions. That makes D wrong.
I know that one of the ideas in paragraph 2 is how “the repression in which the feminine novel was situated also forced women to find innovative and covert ways to dramatize the inner life.” Brontë’s novel is an example of this: “the mad wife locked in the attic symbolizes the passionate and sexual side of Jane’s personality, an alter ego that her upbringing, her religion, and her society have commanded her to incarcerate.” So I’m looking for an answer which mentions that Victorian female writers often had to find new and hidden ways to talk about passions and desires.
A matches some claims in paragraph 1, but the description of a “mad wife locked in the attic” is a “passionate” character, so A can’t be right.
This is discussed in the third paragraph, but not in relation to Brontë’s novel, which is mentioned in the second. So B has to be wrong.
C is correct. It matches with my prediction that the inner lives of women had to be depicted in “covert” ways.
While paragraph 1 mentions the ways the literary world punished coarseness, the word and concept don’t ever make it into the description of Brontë’s novel, so D has to be wrong.
To answer this question, I need to first confirm that the claim in the question stem is supported. If it isn’t, there will likely be an answer choice which states that. If it is, I will also have to determine how it is supported. I know that the critical objections to feminine literature all come in the first paragraph. A quick scan points me to the middle of the paragraph, where the author lists how critics objected “more generally to the moral tone of a work, such as the ‘vein of perilous voluptuousness’ one alert critic detected in Adam Bede.” So I’m probably looking for an answer choice which states that the claim is supported with a critical opinion about Adam Bede.
The claim is definitely supported with a quote from a critic, so A has to be wrong.
I can immediately eliminate B, because the first word of this answer choice should be “Yes.”
Aurora Leigh is mentioned in the first half of the sentence which mentions moral tone. But the “or” between them suggests that Aurora Leigh may be part of a related, but still different, point. That point is more obvious when the start of the sentence is read: “It [coarseness] could refer to the ‘damns’ in Jane Eyre, the dialect in Wuthering Heights, the slang of Rhoda Broughton’s heroines, the colloquialisms in Aurora Leigh, or more generally to the moral tone of a work.” That suggests that the reference to Aurora Leigh is a different example of coarseness: one that does not necessarily refer to the general moral tone of a work. C therefore has to be wrong.
D, though, matches my prediction, since it mentions a critical opinion about Adam Bede.
The key word in this question is “colloquial,” which means everyday normal speech. Since the question stem uses a lot of words to highlight the class of people speaking, it’s safe to assume that these farmers would speak with coarse language: language that would be unusual to hear from an “English gentlewoman” (paragraph 1). And I know from paragraph 1 that novels accused of coarseness were not looked on well by critics.
I actually don’t have clear criteria in the passage for judging why a novel by a woman would be ignored, so I can safely eliminate A.
B matches my prediction, so it must be correct.
I’ve seen this answer choice in this question set before. The author would praise a feminine novel for intensity and profundity (paragraph 2), but the passage does not suggest that critics would. So C has to be wrong.
Aurora Leigh is listed as a novel that critics disliked because it was coarse, and the novel described in the question stem is likely to be considered coarse. But I don’t see anything in the passage that would help me to decide what makes one novel coarser than the other. D therefore has to be wrong.
The Florence Nightingale quote from paragraph 2 is “Give us back our suffering […] for out of nothing comes nothing. But out of suffering may come the cure. Better have pain than paralysis.” It is also prefaced by a description of the quote: “Florence Nightingale thought the effort of repression itself drained off women’s creative energy.” If Nightingale saw the current status quo as one of paralysis that “drained off women’s creative energy,” then the “pain” that comes from writing more freely would hopefully encourage women’s creative energy.
A matches my prediction, since it implies that giving women more freedom would keep writing from draining women’s creative energy.
B just doesn’t make sense; why would writing more freely make women’s work more restrained? B has to be wrong.
I don’t know what Florence Nightingale thought made a work more or less subversive based on the passage. It just doesn’t come up in her quote, so C has to be wrong.
If anything, Florence Nightingale is suggesting the opposite. If women are more creative, they are likely to also be more innovative.
The whole passage talks about social repression, so I can only answer this question by using the main idea. I know that the main idea is crystallized in paragraph 2: “the repression in which the feminine novel was situated also forced women to find innovative and covert ways to dramatize the inner life, and led to a fiction that was intense, compact, symbolic, and profound.” So I’m looking for a choice which suggests that social repression is both harmful, but can also encourage innovation.
“Invariably” is a very strong claim to make. It’s hard to see how the passage supports it, especially given that the passage does not provide any examples of women authors who went mad because of the repressiveness of the times. The passage only mentions a fictional character who went mad, which is not the same thing. A therefore has to be wrong.
The passage exclusively focuses on female authors, so B is automatically wrong by virtue of mentioning creative men.
C is wrong for a similar reason to B. The passage is focused on women; it only refers to men in order to illustrate how restricted female writers of the time were. Since I have no idea how creative male authors were at the time, I have no way to determine if female authors were more or less creative than they were, so C can’t be correct.
D matches my prediction, insofar as it mentions how hurtful social repression was (harm one artist’s creativity) but also how it could spur innovation (enhancing another’s resourcefulness).
I’m looking for a part of the passage that the quote in the question stem strengthens. The quote describes a female author being accused of thinking in a way that she shouldn’t. To mention “The day dreams in which you habitually indulge” implies that Southey is referring to particular dreams that are harmful to Brontë, which in turn suggests that there might be daydreams that would be less harmful to her. So I’m likely looking for an answer choice which mentions that there were wrong ways for women to think.
Southey’s letter doesn’t mention anything about Brontë’s use of male characters, so A can’t be right.
B is an OK candidate for the right answer, since it does imply that women needed to think in specific ways. But B is also most of the main idea, so I should eliminate C and D just in case they are more directly affected by the quote in the question stem.
Nothing in Southey’s quote refers to innovations in feminine writing. If anything, Southey implies the opposite: that women should not innovate at all, since it sounds like he’s saying they shouldn’t prioritize literature enough to innovate. So C has to be wrong.
All I have is Southey’s quote; I don’t have a clear sense of how Southey’s letter affected Brontë. That means I can’t know how paralyzed she was, or if she suffered refinement or restraint. Therefore, I don’t know enough to choose D, so B must be correct.
While the author mentions Taylor’s book in paragraph 4, the author does not explain what was wrong with it until paragraph 5: “The trouble is that it was taken too far and became self-destructive.” If Taylor’s book were to avoid the negative effects the author attributes to it, then, I can deduce that the author would want Taylor’s book to limit its analysis: to say that what its talking about should not be taken too far.
There’s no obvious way that Taylor’s being a manager would have kept others from taking his work too far, so A has to be wrong.
The author doesn’t imply that Taylor’s abstractness was a problem, so B has to be wrong.
C matches my prediction insofar as it mentions how Taylor just needed to limit his work in some way. So I would choose C and move on.
I have no clear sense of whether Taylor based his work on observation or theory; the passage doesn’t say either. So D has to be wrong for that reason alone.
Roger Smith is used as an example for the idea of that last paragraph. That idea is most clearly stated in the previous paragraph: “With bigger bureaucracies, companies couldn’t respond quickly to market changes, new technology, competitors, or customer needs. The more powerful top executives became, the less they knew.” That suggests that when Roger Smith said he did not know what went wrong at GM, he actually didn’t know.
Confidence is mentioned by Mel Struckey in paragraph 6 as a reason why executives might not have known about their businesses, but the passage does not clearly suggest that Roger Smith suffered from that “phobia of manufacturing,” so I can eliminate A.
There is nothing in the last paragraph which suggests that Smith really knew what went wrong. This is clear in the sentence after Smith’s quote: “To fathom what had gone wrong, Smith would truly have had to understand the way automobiles are designed and made; he apparently never did.” B is therefore incorrect.
Since the passage doesn’t provide the specific question he was asked, there is nothing in the passage to suggest that the question was misunderstood. C has to be wrong.
D is the last choice, and it matches my prediction, so it has to be correct.
The author’s main idea is that the notion of management as a science has failed. So the author would therefore likely object to business school students trying to learn about management techniques.
The author would actually like this, since in paragraph 3, the author says that the most successful executives “have been semi-fanatics who doggedly pursued a few good ideas. People like Sam Walton (Wal-Mart), Ray Kroc (McDonald’s), William McGowan (MCI), and Bill Gates (Microsoft).” Each of those executives were deeply immersed in their industry; such a list suggests that A has to be wrong.
B has to be correct. The author would definitely object to students learning about what the passage directly and vehemently argues against.
I don’t really know how to relate C to the passage argument, so C has to be wrong.
This is far too practical for the passage, which speaks about management theory and approaches to understanding the task of management. D is therefore also wrong.
The specific quote referenced in the question stem is “As a society, we have spent more than a decade paying for mistakes like Smith’s.” This most immediately refers to how uninformed Smith was about his industry: a point made in the previous paragraph that Smith is the example for. But since this example comes up at the end, it is also likely an illustration of the author’s main point: that approaching management as a science or as a general skill, independent of industry, is bad. Either of those will make for a good right answer.
A actually mentions both of my predictions, so A has to be right.
The last paragraph only mentions Smith’s lack of knowledge. It doesn’t give me enough information regarding how Smith actually ran GM for me to know if he relied too much on a large staff, or his own judgments. That makes B wrong.
For C to be right, the last paragraph would have to refer to some number of cases. Otherwise, how would I know that those cases were critical, or that his poor judgment affected more than 50% of cases? Since it does not do so, C has to be wrong.
The passage does not suggest that Smith was an inexperienced manager, and I have no clue what consumer needs he failed to address, so D has to be wrong.
The passage is mostly critical of management practices, so I know one possible right answer could mention something about avoiding hiring people who rely on management practices. But the passage only praises some businesses in one part of the passage: paragraph 3, where the author brings up the most successful executives “have not been all-purpose executives, casually changing jobs and succeeding on the strength of dazzling analysis. Instead, they have been semi-fanatics who doggedly pursued a few good ideas.” So the right answer could also perhaps mention having an executive who was fanatical about hand-held calculators.
I don’t see anything in the passage about talking to other successful entrepreneurs or learning from other businesses, so I can safely eliminate A.
B confuses me. Why would making a product similar to another product make a business more successful? B is so random that it has to be wrong.
The passage is focused on management, so the right answer has to be about executives, and not about those that executives hire. That’s enough to make C correct.
D has to be the right answer. It mentions management getting more into their industry, which matches my prediction.
Large staffs are mentioned as part of the problem with management in paragraph 6: “With bigger bureaucracies, companies couldn’t respond quickly to market changes, new technology, competitors, or customer needs.” The more powerful top executives became, the less they knew.” So I’m looking for an answer which suggests that a physician with a large staff might be more out of touch with patients, or with medical knowledge.
Responsibility is not really discussed in the passage, so I can eliminate A.
B matches my prediction: that the physician would be more ignorant, just as the executive with a larger staff knew less. B therefore has to be right.
The passage doesn’t describe trying to stop problems versus trying to cure them, so C has to be wrong.
I know that executives with large staffs may know less, but I don’t see anything in the passage which suggests that they do less. So D has to be wrong.
Analysis in the passage is persistently associated with the science of management that the author hates, so I’m looking for an answer choice which is clearly negative.
A is pretty negative; to call something “superficial” is usually an insult. But since my prediction is so broad, it makes sense to just check and make sure there are no other negative answer choices, because if there are, I’ll need to make a choice between them.
B is a pretty positive term, so I can eliminate it.
C is also quite positive; we tend to think well of people who want to improve.
And D is neutral at worst and positive at best. Being competitive is often celebrated in the business world. D is certainly nowhere near as negative as A is, so A has to be right.
I’m looking for a claim with no evidence to support it. Since it’s very difficult to know when nothing happens, my best strategy here is to check and see if each answer choice is supported in the passage.
The end of paragraph 1, where the “death of management” is first mentioned, supports A with a list of companies that have suffered because of bad management: “Sears, Westinghouse, and IBM.” So since A is supported, it can’t be correct.
The cost of bad management to society is brought up in the last paragraph, where it is explained: “As a society, we have spent more than a decade paying for mistakes like Smith’s. Inept management, though not the only cause of corporate turmoil, has been a major contributor. ‘Downsizing’ and ‘restructuring’ are but the catchwords for the harsh process by which companies are seeking to regain their edge.” Since “downsizing” and “restructuring” are implied to be societal consequences for poor management, B is supported enough to be incorrect.
This claim is made in the first paragraph, but I don’t see any proof of it. C also makes a good candidate for the right answer because it’s actually a really difficult claim to support. The only way I could imagine supporting C would be by explaining why the idea of management had to occur in America, which might mean explaining how management is seen in other countries. Since nothing like that occurs in the passage, C must be correct.
One whole paragraph is dedicated to Taylor—a paragraph that includes quotes—so it is a pretty safe bet that D is supported in the passage. That makes D wrong.
The passage begins by positing a problem: that people may think criticism is extraneous, just a parasite to art. The author then explains the value of criticism. So I’m looking for an answer which claims that criticism is a good and necessary discipline that is not a parasite to art: that is its own independent and valuable discipline.
This is where the passage starts, but the author argues against this view, so A can’t be correct.
This claim is made in paragraph 4, and does help explain why criticism has to exist. However, I know it can’t be the main idea because B is introduced as “another reason why criticism has to exist.” In other words, B is one of a few pieces of evidence for a claim besides the main idea. The main point of a passage has to be the point that most of the evidence in the passage supports, so B has to be wrong.
The author says the opposite in paragraph 3: that without criticism, a culture risks losing its cultural memory. Since C distorts the passage, it has to be wrong.
D is the last answer choice, and it encapsulates the passage argument well. It implicitly explains that criticism is an independent discipline, and one that deserves to exist. Specifically, criticism helps “educate the public about art” in two ways: by helping to shape cultural tradition through education (paragraph 3), and by helping art to speak (last paragraph).
This phrase occurs in the last paragraph, where the author states his suspicion of poetry that “speaks,” or has a message that goes beyond its art. Paragraph 4 states that “Poetry is a disinterested use of words: it does not address a reader directly. When it does so, we usually feel that the poet has some distrust in the capacity of readers and critics to interpret the poet’s meaning without assistance and has therefore dropped into the subpoetic level of metrical talk (‘verse’ or ‘doggerel’), which anybody can learn to produce.” So I know that the right answer has to mention that metrical talk describes poetry the author does not like: that tries to explain the poetry, since it implies that the poet does not trust readers and thinkers to read the poem on their own.
Rhyming isn’t mentioned in that last paragraph, so I can eliminate A.
The last paragraph suggests the opposite: that poetry with metrical talk will be more understandable by the average reader than normal poetry. That makes B incorrect.
C matches my prediction: that poetry with metrical talk addresses the reader directly, unlike normal poetry which does not. C is therefore correct.
The last paragraph does not really focus on traditions, so D has to be incorrect.
One way to approach this question is to just go through the answer choices and pick the answer that is supported with an example from the passage. The wrong answers will not be supported: will just be stated, with no other effort by the author to convince readers of the point. Or, I can try to locate the examples in the passage. One prominent example is in paragraph 3: where the author argues that “it is clearly the simple truth that there is no real correlation either way between the merits of art and its public reception” by providing the examples of Shakespeare and Keats. So I can expect that the answer will reference paragraph 3.
This stated in the first paragraph, but it is immediately followed by another claim: “criticism is evidently something of an art too.” That means that there is no room for the author to support the point. A is therefore wrong.
This is stated in paragraph 2, which I know contains no examples, so I can easily eliminate B.
C matches the claim from paragraph 3 which I know is supported by an example, so C is correct.
This is also in paragraph 2, where there are no examples, so D has to be wrong.
This speaks to the author’s argument in paragraph 3 and 4, and so the main idea: that criticism educates people with regards to how to approach art. The author would therefore probably judge the person at the poetry reading as making an uneducated comment: as perhaps needing to wait until they encountered some criticism on the poetry.
A matches my prediction: that the person in the question stem is not allowing for the place of criticism, and so may not be right about the poetry at the reading. I can therefore choose A confidently.
This is incorrect. The author makes room for subjective and personal responses to poetry, but there are also aspects of poetry that criticism is meant to interact with that aren’t just individual responses. Since poetry is not “purely” subjective and personal then, B has to be incorrect.
The author never says that the public is usually wrong about art. Paragraph 3 actually argues the opposite: that “there is no real correlation either way between the merits of art and its public reception.” In other words, the public is neither usually right or wrong about art, which makes C correct.
The author would disagree with D, especially because of the last paragraph, where the author argues that “Criticism can talk, and all the arts are dumb.” Poets, in other words, can’t set up the standards for artistic success, making D wrong.
I know the value of criticism to society because it is implied by the end of paragraph 3: “A public that tries to do without criticism, and asserts that it knows what it wants or likes, brutalizes the arts and loses its cultural memory.” If that’s what happens when criticism does not occur, then criticism must keep people from brutalizing the arts, and help to retain cultural memory.
Nothing in the passage talks about criticism helping people to create art. The passage mainly focuses on their being artists, the public, and critics, not how one becomes the other, or becomes better at being one of those options.
B perfectly matches my prediction from paragraph 3, so I can choose it and move on.
Paragraph 3 says the opposite: that a public without criticism brutalizes the arts, which implies that critics keep the public from doing C. So C has to be wrong.
The passage doesn’t talk about aesthetic theories, so I have no reason to see D as correct.

Leave a Reply

Your email address will not be published. Required fields are marked *